UNIT 2 more questions

¡Supera tus tareas y exámenes ahora con Quizwiz!

The nurse administered 12 units of regular insulin to the patient with Type 1 dia- betes at 0700. Which meal would prevent the client from experiencing hypoglyce- mia? 1. Breakfast. 2. Lunch. 3. Supper. 4. HS snack.

1 1. Regular insulin peaks in 2-4 hours; therefore, the breakfast meal would prevent the client from developing hypoglycemia. 2. Lunch would cover a 0700 dose of Humulin N, an intermediate-acting insulin. 3. Supper would cover a 1600 dose of Humulin R, a short-acting insulin. 4. The HS (nighttime) snack would cover a 1600 dose of Humulin N, an intermediate- acting insulin.

The client with the flu has been taking acetylcysteine (Mucomyst), a mucolytic. Which adverse effect would the nurse assess for? 1. Bronchospasm. 2. Nausea. 3. Fever. 4. Drowsiness.

1 1. Mucomyst can cause bronchospasm, which will impair the client's breathing, not improve it. An adverse reaction is a reason to immediately discontinue the medication. 2. Nausea is a side effect of many medications and can usually be managed by taking the medication with food. A side effect is not an adverse effect. 3. Fever would result from the cold, flu, or infection, not from the medication. 4. Drowsiness is caused by some cold and flu preparations, usually the antihistamines. Mucomyst causes the client to expectorate secretions, which will keep the client awake.

Which electrolyte replacement should the nurse anticipate being ordered by the health-care provider in the client diagnosed with DKA who has just been admitted to the ICU? 1. Glucose. 2. Potassium. 3. Calcium. 4. Sodium.

2 1. Glucose is elevated in DKA; therefore, the HCP would not be replacing glucose. 2. The client in DKA loses potassium from increased urinary output, acidosis, cata- bolic state, and vomiting. Replacement is essential for preventing cardiac dys- rhythmias secondary to hypokalemia. 3. Calcium is not affected in the client with DKA. 4. The prescribed IV for DKA—0.9% normal saline—has sodium, but it is not specifically ordered for sodium replacement. This is an isotonic solution.

Which assessment data indicate the client diagnosed with diabetic ketoacidosis is responding to the medical treatment? 1. The client has tented skin turgor and dry mucous membranes. 2. The client is alert and oriented to date, time, and place. 3. The client's ABG results are pH 7.29, PaCO2 44, HCO3 15. 4. The client's serum potassium level is 3.3 mEq/L.

2 1. This indicates the client is dehydrated, which does not indicate the client is get- ting better. 2. The client's level of consciousness can be altered because of dehydration and acidosis. If the client's sensorium is intact, the client is getting better and responding to the medical treatment. 3. These ABGs indicate metabolic acidosis; therefore, the client is not responding to treatment. 4. This potassium level is low and indicates hypokalemia, which shows the client is not responding to medical treatment.

The emergency department nurse is caring for a client diagnosed with HHNS who has a blood glucose of 680 mg/dL. Which question should the nurse ask the client to determine the cause of this acute complication? 1. "When is the last time you took your insulin?" 2. "When did you have your last meal?" 3. "Have you had some type of infection lately?" 4. "How long have you had diabetes?"

3 1. A client with type 2 diabetes usually is prescribed oral hypoglycemic medications, not insulin. 2. The client could not eat enough food to cause a 680-mg/dL blood glucose level; therefore, this question does not need to be asked. 3. The most common precipitating factor is infection. The manifestations may be slow to appear, with onset ranging from 24 hours to 2 weeks. 4. This does not help determine the cause of this client's HHNS.

The nurse is assessing the feet of a client with long-term type 2 diabetes. Which assessment data warrant immediate intervention by the nurse? 1. The client has crumbling toenails. 2. The client has athlete's foot. 3. The client has a necrotic big toe. 4. The client has thickened toenails.

3 1. Crumbling toenails indicate tinea unguium, which is a fungus infection of the toenail. 2. Athlete's foot is a non-life-threatening fungal infection. 3. A necrotic big toe indicates "dead" tis- sue. The client does not feel pain, does not realize the injury, and does not seek treatment. Increased blood glucose levels decrease the oxygen supply needed to heal the wound and increase the risk for developing an infection. 4. Big, thick toenails are fungal infections and do not require immediate intervention by the nurse.

The nurse is preparing to administer warfarin (Coumadin), an anticoagulant. The client's current laboratory values are as follows: PT 22 PT 39 Control 12.9 Control 36 INR 2.6 Which action should the nurse implement? 1. Question administering the medication. 2. Prepare to administer AquaMEPHYTON (vitamin K). 3. Notify the health-care provider to increase the dose. 4. Administer the medication as ordered.

4 1. The INR is within therapeutic range; therefore, the nurse should not question administering this medication. 2. Vitamin K is the antidote for Coumadin toxicity, but the client is not in a toxic state. 3. There is no reason to notify the HCP to request an increase in the dose because the client is in the therapeutic range. 4. When the nurse is administering Coumadin, the International Normalized Ratio (INR) must be monitored to determine therapeutic level, which is 2-3. Because the INR is 2.6, the nurse should administer this medication. MEDICATION MEMORY JOGGER: When trying to remember which laboratory value correlates with which anticoagulant, here's a helpful hint: "PT boats go to war (warfarin), and if you cross the small 't's' in 'Ptt' with one line it makes an 'h' (heparin)."

The female client is being admitted to a medical unit with a diagnosis of pneumonia. Which intervention would the nurse implement? Rank in order of implementation. 1. Start an intravenous access line. 2. Administer the IVPB antibiotic. 3. Teach to notify the nurse of any vaginal itching. 4. Obtain sputum and blood cultures. 5. Place an identity band on the client.

5, 4, 1, 2, 3 5. The laboratory technician that will draw the blood cultures will need the band to identify the client before draw- ing the specimen, and the nurse will need the band before administering the medication. Checking for the right client is one of the five rights of medication administration. 4. Cultures are obtained prior to the initiation of antibiotics to prevent skewing of the results. 1. An intravenous line must be initiated before the nurse can administer IV medications. 2. Intravenous antibiotics should be administered within 1-2 hours of the order being written. This should always be considered a "now" medica- tion. 3. Superinfections are a potential compli- cation of antibiotic therapy. Vaginal yeast infections occur when the good bacteria are killed off by the antibiotic. Diarrhea from destruction of intestinal flora is also a possibility.

The client is receiving an intravenous infusion of heparin. The bag hanging has 25,000 units of heparin in 500 mL of D5W at 14 mL per hour via an intravenous pump. How many units of heparin is the client receiving every hour? Answer ____________________

700 units of heparin are being infused every hour. When determining the units, the nurse must first determine how many units are in each milliliter. 25, 5 0 0 0 0 0 m U L nits 50 units per mL50 units per mL 14 mL per hour 700 mL per hour

The client is diagnosed with a pulmonary embolus and is receiving a heparin drip. The bag hanging is 20,000 units/500 mL of D5W infusing at 22 mL/hr. How many units of heparin is the client receiving each hour? ________

880 units. If there are 20,000 units of heparin in 500 mL of D5W, there are 40 units in each mL: 20,000 ÷ 500 = 40 units If 22 mL are infused per hour, then 880 units of heparin are infused each hour: 40 × 22 = 880

The client diagnosed with restrictive airway disease (asthma) has been prescribed a glucocorticoid inhaled medication. Which information should the nurse teach regarding this medication? 1. Do not abruptly stop taking this medication; it must be tapered off. 2. Immediately rinse the mouth following administration of the drug. 3. Hold the medication in the mouth for 15 seconds before swallowing. 4. Take the medication immediately when an attack starts.

2 1. This applies to systemically administered steroids, not to inhaled steroids. 2. The steroids must pass through the oral cavity before reaching the lungs. Allowing the medication to stay within the oral cavity will suppress the normal flora found there, and the client could develop a yeast infection of the mouth (oral candidiasis). 3. Holding the medication in the mouth increases the risk of an oral yeast infection, and the medication is inhaled, not swallowed. 4. Inhaled steroids are not used first; the beta-adrenergic inhalers are used for acute attack.

A client diagnosed with diabetes mellitus is on an insulin infusion drip. The insulin bag indicates there are 100 units of insulin in 1,000 milliliters (mL) of normal saline. Based on the client's blood glucose reading, the client should receive 1.5 units per hour. To ensure that the client receives 1.5 units per hour, the nurse should set the pump at ______ mL/hr.

ANSWER: 15 100 units : 1,000 mL :: 1.5 units : X mL 100X = 1,500 X = 15

A client, hospitalized for a severe case of pneumonia, is asking a nurse why a sputum sample is needed. The nurse should reply that the primary reason is to: 1. complete the first of three samples to be collected. 2. differentiate between pneumonia and atelectasis. 3. encourage expectoration of secretions. 4. help select the appropriate antibiotic.

ANSWER: 4 Culturing the causative organism and testing sensitivities for the most effective antibiotic is the main reason that a sample is collected. Three samples are taken for a client with suspected tuberculosis. A client with at- electasis may get pneumonia, but generally this is not a test used to diagnose atelectasis. Although secretions are expectorated to obtain a sputum sample, the collection itself does not encourage future expectoration of secretions.

The client is receiving an intravenous infusion of heparin. The bag hanging has 40,000 units of heparin in 500 mL of D5W. The HCP has ordered the medication to be delivered at 1200 units per hour. At what rate would the nurse set the intravenous pump? Answer ____________________

15 mL per hour. When setting the intra- venous pump, the nurse must first determine the number of units per milliliter. 40 5 ,0 0 0 0 0 m u L nits 80 units per mL 15 mL 1200 units per hour 80 units per mL

The nurse is preparing to administer an IVPB antibiotic to a client diagnosed with pneumonia; 10 mL of the medication is mixed in 100 mL of saline. At what rate would the nurse set the pump to infuse the medication in 30 minutes? Answer ____________________

220 mL/hour. The nurse should set the pump at 220 mL/ hour. Pumps are set at an hourly rate. 60 minutes divided by 30 equals 2. 100 10 110 110 multiplied by 2 220.

The client diagnosed with type 1 diabetes is receiving Humalog, a rapid-acting insulin, by sliding scale. The order reads blood glucose level: <150, zero (0) units; 151 to 200, three (3) units; 201 to 250, six (6) units; >251, contact health-care provider. The unlicensed assistive personnel (UAP) reports to the nurse the client's glucometer reading is 189. How much insulin should the nurse administer to the client? _________

Three (3) units. The client's result is 189, which is between 151 and 200, so the nurse should administer 3 units of Humalog insulin subcutaneously.

The client newly diagnosed with Type 1 diabetes asks the nurse, "Why should I get an external portable insulin pump?" Which statement is the nurse's best response? 1. "It will cause you to have fewer hypoglycemic reactions and it will control blood glucose levels better." 2. "Insulin pumps provide an automatic memory of the date and time of the last 24 boluses." 3. "The pump injects intermediate-acting insulin automatically into the vein to maintain a normal blood glucose level." 4. "The portable pump is the easiest way to administer insulin to someone with Type 1 diabetes and is highly recommended."

1 1. A portable insulin pump is a battery- operated device that uses rapid-acting insulin—Lispro, Humalog, or NovoLog. It delivers both basal insulin infusion (continuous release of a small amount of insulin) and bolus doses with meals. This provides fewer hypoglycemic reactions and better blood glucose levels. 2. The pumps do provide a memory of boluses, but that is not the nurse's best response to explain why a client should get an external portable insulin pump. 3. External portable insulin pumps are only used to deliver rapid-acting insulin subcuta- neously. Intermediate- and long-acting insulins are not used with an external portable insulin pump because of unpre- dictable control of blood glucose. 4. The insulin pump is not recommended as the initial way to administer insulin because the success of the insulin pump depends on the client's knowledge and compliance. Initially most clients start injecting insulin with a syringe and then graduate to the pumps.

The home health nurse is completing the admission assessment for a 76-year-old client diagnosed with type 2 diabetes controlled with 70/30 insulin. Which intervention should be included in the plan of care? 1. Assess the client's ability to read small print. 2. Monitor the client's serum PT level. 3. Teach the client how to perform a hemoglobin A1c test daily. 4. Instruct the client to check the feet weekly.

1 1. Age-related visual changes and diabetic retinopathy could cause the client to have difficulty in reading and drawing up insulin dosage accurately. 2. The PT level is monitored for clients receiving Coumadin, an anticoagulant, which is not ordered for client with diabetes, type 1 or 2. 3. Glycosylated hemoglobin is a serum blood test usually performed in a laboratory, not in the client's home. The hemoglobin Alc is performed every three (3) months. Self- monitoring blood glucose (SMBG) should be taught to the client. 4. The client's feet should be checked daily, not weekly. In a week the client could have developed gangrene from an injury the client did not realize he or she had.

The nurse on a medical unit is administering 0900 medications. Which medication should the nurse question administering? 1. Acetylcysteine (Mucomyst), a mucolytic, to a client who is coughing forcefully. 2. Cefazolin (Ancef), an antibiotic, IVPB to a client diagnosed with the flu. 3. Diphenhydramine (Benadryl), an antihistamine, to a client who is congested. 4. Dextromethorphan (Robitussin), an antitussive, to a client who has pneumonia.

1 1. An adverse effect of Mucomyst is bron- chospasm. This client should be assessed for bronchospasm before administering a dose of Mucomyst. 2. Antibiotics are frequently administered to clients with viral infections to prevent secondary bacterial infections. This client is considered at risk or the client would not be in a hospital receiving care. There is no reason to question this medication. 3. Antihistamines are prescribed for conges- tion; there is no reason to question this medication. 4. A symptom of pneumonia is a cough. There is no reason to question this medication.

The overweight client diagnosed with Type 2 diabetes reports to the clinic nurse that he has lost 35 pounds in the last 4 months. Which action should the nurse imple- ment first? 1. Determine if the client has had an increase in hypoglycemic reactions. 2. Instruct the client to make an appointment with the health-care provider. 3. Ask the client if he has been trying to lose weight or has it happened naturally. 4. Check the client's last weight in the chart with the weight obtained in the clinic.

1 1. Changes in weight will affect the amount of medication needed to control blood glucose. The nurse should determine if the client's medica- tion dose is too high by determining if the client has had an increase in hypo- glycemic reactions. This is the nurse's first intervention. 2. A significant weight loss may require a decrease or discontinuation of oral hypo- glycemic medication, but the nurse should first determine if the client has had symp- toms of hypoglycemia before referring him or her to the HCP. 3. Determining if the client was deliberately losing weight or was losing without trying is significant because a 35-pound weight loss in 4 months would warrant interven- tion, depending on what caused the weight loss. However, this should not be the nurse's first intervention. 4. The nurse should confirm the client's weight loss with the clinic scale and the last weight in the client's chart, but it is not the clinic nurse's first intervention. MEDICATION MEMORY JOGGER: Remem- ber that the first step in the nursing process is assessment. Words such as check, monitor, determine, ask, take, auscultate, and palpate indicate that the nurse is assessing the client. Assessment should be done before implementing an independent nursing action or notifying the health-care provider, except in certain serious or life-threatening situations.

The client diagnosed with adult respiratory distress syndrome (ARDS) has been found to have a disease-causing organism resistant to the antibiotics being given. Which intervention should the nurse implement? 1. Monitor for therapeutic blood levels of the aminoglycoside antibiotic prescribed. 2. Prepare to administer the glucocorticoid medication ordered intramuscularly. 3. Obtain an order for repeat cultures to confirm the identity of the resistant organism. 4. Place the client on airborne isolation precautions.

1 1. Currently the medications used to treat resistant bacteria are the amino- glycoside antibiotics. Vancomycin is the drug of choice, but gentamycin may also be used. These medications can be toxic to the auditory nerve and to the kidneys. The therapeutic range is 10-20 mg/dL. The nurse should monitor the blood levels. 2. If ordered, the steroid would be given intravenously, not intramuscularly. 3. The culture does not need to be repeated; this would add unnecessary expense to the client. 4. The client should be placed on contact and possibly droplet precautions. Airborne isolation is required for tuberculosis.

The client with the flu is prescribed the over-the-counter cough suppressant dextro- methorphan. Which information should the nurse teach regarding this medication? 1. Take the medication every 4-8 hours as needed for cough. 2. The medication can cause addiction if taken too long. 3. Do not drive or operate machinery while taking the drug. 4. Do not take a beta blocker while taking this medication.

1 1. Dextromethorphan is relatively safe in the recommended dose range of 10-30 mg every 4-8 hours. At these levels it does not produce respiratory depression and side effects are not common. 2. This medication does not have the potential to cause addiction. 3. This medication does not produce drowsi- ness, so driving or operating machinery while taking dextromethorphan is accept- able. 4. The medication does not slow the heart rate, and there is no reason for a client not to take a prescribed beta blocker medication while taking dextromethorphan.

Which information should the nurse teach the client who is prescribed a glucocorti- coid inhaler? 1. Advise the client to gargle after each administration. 2. Instruct the client to use the inhaler on a PRN basis. 3. Encourage the client not to use a spacer when using the inhaler. 4. Teach the client to check his or her forced expiratory volume daily.

1 1. Gargling after each administration will help decrease the development of oropharyngeal yeast infections. 2. Glucocorticoids are intended for preven- tive therapy, not for aborting an ongoing asthma attack, and they should not be taken on a PRN basis. 3. A spacer, a device that attaches directly to the metered-dose inhaler, should be used because a spacer increases the delivery of the drug to the lungs and decreases depo- sition of the drug on the oropharyngeal mucosa. 4. Forced expiratory volume (FEV) is the single most useful test of lung function, but the instrument required is a spirome- ter, which is expensive, cumbersome, and not suited for home use.

The nurse administered 28 units of Humulin N, an intermediate-acting insulin, to a client diagnosed with type 1 diabetes at 1600. Which intervention should the nurse implement? 1. Ensure the client eats the bedtime snack. 2. Determine how much food the client ate at lunch. 3. Perform a glucometer reading at 0700. 4. Offer the client protein after administering insulin.

1 1. Humulin N peaks in 6 to 8 hours, mak- ing the client at risk for hypoglycemia around midnight, which is why the client should receive a bedtime snack. This snack will prevent nighttime hypoglycemia. 2. The food intake at lunch will not affect the client's blood glucose level at midnight. 3. The client's glucometer reading should be done around 2100 to assess the effectiveness of insulin at 1600. 4. Onset of Humulin N, an intermediate- acting insulin, is 2 to 4 hours but it does not peak until 6 to 8 hours.

The client diagnosed with the flu is prescribed the cough medication hydrocodone. Which information should the nurse teach the client regarding this medication? 1. Teach the client to monitor the bowel movements for constipation. 2. Driving or operating machinery is all right while taking this medication. 3. This medication usually causes insomnia, so plan for rest periods. 4. This medication is more effective when taken with a mucolytic.

1 1. Hydrocodone is an opioid and can slow the peristalsis of the bowel, resulting in constipation. The client should be aware of this and increase the fluid intake and use bulk laxatives and stool softeners, if needed. 2. Opioids can cause drowsiness, so driving or operating machinery should be discour- aged. 3. Opioids usually cause the client to be drowsy, not have insomnia. 4. Hydrocodone is a cough suppressant and a mucolytic is an expectorant. These are opposite-acting medications.

The client diagnosed with a massive pulmonary embolus is ordered the thrombolytic streptokinase. The nurse notes on the Medication Administration Record that the client is allergic to the "-mycin" medications, including streptomycin. Which action should the nurse implement? 1. Call the HCP to report the allergy. 2. Administer the medication as ordered. 3. Call the pharmacist to substitute medication. 4. Check the bleeding-time laboratory values.

1 1. Streptokinase is a foreign protein extracted from the cultures of strepto- cocci bacteria, and streptomycin is derived from Streptomyces. As a result, this could possibly cause the client to have an allergic reaction. The nurse should discuss this allergy with the HCP. 2. The nurse should not administer this medication until determining if the client is at risk for an allergic reaction. 3. The pharmacist is not licensed to change an HCP order. 4. Bleeding times could be assessed after it is determined that the streptokinase will not cause the client to have an allergic reac- tion.

Which assessment data best indicates the client with reactive airway disease has "good" control with the medication regimen? 1. The client's peak expiratory flow rate (PEFR) is greater than 80% of his or her personal best. 2. The client's lung sounds are clear bilaterally, both anterior and posterior. 3. The client has only had three acute exacerbations of asthma in the last month. 4. The client's monthly serum theophylline level is 18 g/mL.

1 1. The PEFR is defined as the maximal rate of airflow during expiration in a relatively inexpensive, handheld device. If the peak flow is less than 80% of personal best, more frequent monitor- ing should be done. The PEFR should be measured every morning. 2. A normal respiratory assessment does not indicate that the medication regimen is effective and has "good" control. 3. Three asthma attacks in the last month would not indicate the client has "good" control of the reactive airway disease. 4. A serum theophylline level between 10 and 20 g/mL indicates the medication is within the therapeutic range, but it is not the best indicator of the client's control of the signs or symptoms.

The client with reactive airway disease is taking the oral sympathomimetic bron- chodilator metaproterenol (Alupent) three times a day. Which intervention should the nurse implement? 1. Instruct the client to take the last dose a few hours before bedtime. 2. Teach the client to decrease the fluid intake when taking this medication. 3. Have the client demonstrate the correct way to use the inhaler. 4. Encourage the client to take the medication with an antacid.

1 1. The client should take the last dose a few hours before bedtime so that the medication does not produce insomnia. 2. The client should increase fluid intake, especially water, because it will make the mucus thinner and help the medication work more effectively. 3. This medication is taken orally; therefore, there is no reason for the client to demon- strate the correct way to use an inhaler. 4. Antacids decrease the absorption of medication; therefore, the medication should not be taken with or within 2 hours of taking an antacid.

The elderly client is admitted to the intensive care department diagnosed with severe HHNS. Which collaborative intervention should the nurse include in the plan of care? 1. Infuse 0.9% normal saline intravenously. 2. Administer intermediate-acting insulin. 3. Perform blood glucometer checks daily. 4. Monitor arterial blood gas results.

1 1. The initial fluid replacement is 0.9% normal saline (an isotonic solution) intravenously, followed by 0.45% saline. The rate depends on the client's fluid volume status and physical health, especially of the heart. 2. Regular insulin, not intermediate, is the insulin of choice because of its quick onset and peak in two (2) to four (4) hours. 3. Blood glucometer checks are done every one (1) hour or more often in clients with HHNS who are receiving regular insulin drips. 4. Arterial blood gases are not affected in HHNS because there is no breakdown of fat resulting in ketones leading to meta- bolic acidosis.

The nurse is teaching the client with Type 1 diabetes how to use an insulin pen injec- tor. Which information should the nurse discuss with the client? 1. Instruct the client to dial in the number of insulin units needed to inject. 2. Demonstrate the proper way to draw up the insulin in an insulin syringe. 3. Discuss that the insulin pen injector must be used in the abdominal area only. 4. Explain that the traditional insulin syringe is less painful than the injector pen.

1 1. The insulin pen injector resembles a fountain pen. It contains a disposable needle and insulin-filled cartridge. When the client operates the insulin pen, the correct dose is obtained by turning the dial to the number of insulin units needed. 2. The insulin pen injector does not require drawing up insulin in a syringe. 3. The insulin pen injector can be used in any subcutaneous site that traditional insulin can be injected. 4. Most clients state that there is less injection pain associated with the insulin pen than with the traditional insulin syringe.

The client diagnosed with type 1 diabetes is found lying unconscious on the floor of the bathroom. Which intervention should the nurse implement first? 1. Administer 50% dextrose IVP. 2. Notify the health-care provider. 3. Move the client to the ICU. 4. Check the serum glucose level.

1 1. The nurse should assume the client is hypoglycemic and administer IVP dex- trose, which will rouse the client imme- diately. If the collapse is the result of hyperglycemia, this additional dextrose will not further injure the client. 2. The health-care provider may or may not need to be notified, but this is not the first intervention. 3. The client should be left in the client's room, and 50% dextrose should be administered first. 4. The serum glucose level requires a venipuncture, which will take too long. A blood glucometer reading may be ob- tained, but the nurse should first treat the client, not the machine. The glucometer only reads "low" after a certain point, and a serum level is needed to confirm exact glucose level.

The nurse is discussing the oral hypoglycemic medication Micronase with the client diagnosed with Type 2 diabetes. Which information should the nurse discuss with the client? 1. Instruct the client to take the oral hypoglycemic medication with food. 2. Explain that hypoglycemia will not occur with oral medications. 3. Tell the client to notify the HCP if a headache, nervousness, or sweating occurs. 4. Recommend the client check the ketones in the urine every morning.

1 1. The oral hypoglycemic medication should be administered with food to decrease gastric upset. 2. The client receiving oral hypoglycemic medications can experience hypoglycemic reactions, as can clients receiving insulin. 3. These are signs or symptoms of hypo- glycemia, and the client should be able to treat this without notifying the health-care provider. 4. Ketones are a byproduct of the breakdown of fats, which usually does not occur in clients with Type 2 diabetes because the client has enough insulin to prevent break- down of fats but not enough to keep the blood glucose level within an acceptable level.

The nurse is developing a care plan for the client diagnosed with type 1 diabetes. The nurse identifies the problem "high risk for hyperglycemia related to noncompliance with the medication regimen." Which statement is an appropriate short-term goal for the client? 1. The client will have a blood glucose level between 90 and 140 mg/dL. 2. The client will demonstrate appropriate insulin injection technique. 3. The nurse will monitor the client's blood glucose levels four (4) times a day. 4. The client will maintain normal kidney function with 30-mL/hr urine output.

1 1. The short-term goal must address the response part of the nursing diagnosis, which is "high risk for hyperglycemia," and this blood glucose level is within acceptable ranges for a client who is noncompliant. 2. This is an appropriate goal for a knowledge- deficit nursing diagnosis. Noncompliance is not always the result of knowledge deficit. 3. The nurse is implementing an interven- tion, and the question asks for a goal which addresses the problem of "high risk for hyperglycemia." 4. The question asks for a short-term goal and this is an example of a long-term goal.

The nurse is discussing storage of insulin vials with the client. Which statement indi- cates the client understands the teaching concerning the storage of insulin? 1. "I will keep my unopened vials of insulin in the refrigerator." 2. "I can keep my insulin in the trunk of my car so I will have it at all times." 3. "It is all right to put my unopened insulin vials in the freezer." 4. "If I prefill my insulin syringes, I must use them within 1-2 days."

1 1. This statement indicates the client understands the medication teaching. Keeping the insulin in the refrigerator will maintain the insulin's strength and potency. Once the insulin vial is opened it may be kept at room temperature for 1 month. 2. Insulin vials should not be placed in direct sunlight or in a high-temperature area, such as the trunk of a car, because it will lose its strength. 3. Insulin should not be kept in the freezer because freezing will cause the insulin to break down and lose its effectiveness. 4. Prefilled syringes should be stored in the refrigerator and should be used within 1-2 weeks, not 1-2 days.

The nurse is preparing to hang the next bag of heparin. The client's current labora- tory values are as follows: PT 13.4 PTT 92 Control 12.9 Control 36 INR 1 Which intervention should the nurse implement first? 1. Discontinue the heparin infusion. 2. Prepare to administer protamine sulfate. 3. Notify the health-care provider. 4. Assess the client for bleeding.

1 1. This would be the first intervention because the client is above the thera- peutic range. The therapeutic range for heparin is 1.5 to 2.0 times the control, or 54 to 72. The client's PTT of 92 places the client at risk for bleeding. Therefore, the nurse must prevent further infusion of medication. 2. This is the antidote for heparin, but the nurse would not administer this first. Discontinuing the infusion of heparin for a few hours may be sufficient to correct the overdose. 3. The HCP should be notified of the client's situation, but it is not the first interven- tion. 4. Assessment is the first step in the nursing process, but if the client is in "distress" or experiencing a complication, the nurse should first treat the client.

The client is admitted to the ICU diagnosed with DKA. Which interventions should the nurse implement? Select all that apply. 1. Maintain adequate ventilation. 2. Assess fluid volume status. 3. Administer intravenous potassium. 4. Check for urinary ketones. 5. Monitor intake and output.

1,2,3,4,5 1. The nurse should always address the airway when a client is seriously ill. 2. The client must be assessed for fluid volume deficit and then for fluid volume excess after fluid replacement is started. 3. The electrolyte imbalance of primary concern is depletion of potassium. 4. Ketones are excreted in the urine; lev- els are documented from negative to large amount. Ketones should be mon- itored frequently. 5. The nurse must ensure the client's fluid intake and output are equal.

The diabetic educator is teaching a class on diabetes type 1 and is discussing sick-day rules. Which interventions should the diabetes educator include in the discussion? Select all that apply. 1. Take diabetic medication even if unable to eat the client's normal diabetic diet. 2. If unable to eat, drink liquids equal to the client's normal caloric intake. 3. It is not necessary to notify the health-care provider if ketones are in the urine. 4. Test blood glucose levels and test urine ketones once a day and keep a record. 5. Call the health-care provider if glucose levels are higher than 180 mg/dL.

1,2,5 1. The most important issue to teach clients is to take insulin even if they are unable to eat. Glucose levels are in- creased with illness and stress. 2. The client should drink liquids such as regular cola or orange juice, or eat regular gelatin, which provide enough glucose to prevent hypoglycemia when receiving insulin. 3. Ketones indicate a breakdown of fat and must be reported to the HCP because they can lead to metabolic acidosis. 4. Blood glucose levels and ketones must be checked every three (3) to four (4) hours, not daily. 5. The HCP should be notified if the blood glucose level is this high. Regular insulin may need to be prescribed to keep the blood glucose level within acceptable range.

The HCP prescribed amoxicillin/clavulanate (Augmentin), an antibiotic, for a client diagnosed with chronic obstructive pulmonary disease (COPD) who has a cold. Which intervention should the nurse implement? 1. Discuss the prescription with the HCP because antibiotics do not help viral infec- tions. 2. Teach the client to take all the antibiotics as ordered. 3. Encourage the client to seek a second opinion before taking the medication. 4. Ask the client if he or she is allergic to sulfa drugs or shellfish.

2 1. Antibiotics do not treat viral infections, but HCPs will frequently prescribe prophylactic antibiotics for clients with comorbid condi- tions (such as COPD) to prevent a second- ary bacterial infection. 2. Clients prescribed antibiotics should always be taught to take all the medica- tion as ordered to prevent resistant strains of bacteria from developing. 3. There is no reason for a second opinion; this is standard medical practice. 4. This is a penicillin preparation, not a sulfa medication or iodine.

The HCP has ordered streptokinase (Streptase), a thrombolytic, intravenously for the client diagnosed with a pulmonary embolus. The client has intravenous heparin infusing at 1600 units per hour via a 20-gauge angiocath. Which intervention should the nurse implement? 1. Administer the streptokinase via a Y-tubing. 2. Start a second intravenous site to infuse the streptokinase. 3. Discontinue the heparin and infuse streptokinase via the 20-gauge angiocath. 4. Piggyback the streptokinase through the heparin line at the port closest to the client.

2 1. Blood or blood products are the only fluids infused through Y-tubing. 2. Heparin and streptokinase cannot be administered in the same intravenous line because they are incompatible.The nurse must start a second line to administer the streptokinase simulta- neously with the heparin. The nurse does not need an order to do this. 3. The client needs both of these medica- tions; therefore, the nurse cannot discon- tinue the heparin. Streptokinase is a thrombolytic, which will dissolve the clot in the pulmonary artery, but heparin, an anticoagulant, is prescribed to prevent reformation of the clot. 4. Heparin and streptokinase cannot be administered in the same intravenous line because they are incompatible. The nurse must start a second line to administer the streptokinase simultaneously with the heparin. The nurse does not need an order to do this.

The client with type 2 diabetes controlled with biguanide oral diabetic medication is scheduled for a computed tomography (CT) scan with contrast of the abdomen to evaluate pancreatic function. Which intervention should the nurse implement? 1. Provide a high-fat diet 24 hours prior to test. 2. Hold the biguanide medication for 48 hours prior to test. 3. Obtain an informed consent form for the test. 4. Administer pancreatic enzymes prior to the test.

2 1. High-fat diets are not recommended for clients diagnosed with diabetes, and food does not have an effect on a CT scan with contrast. 2. Biguanide medication must be held for a test with contrast medium because it increases the risk of lactic acidosis, which leads to renal problems. 3. Informed consent is not required for a CT scan. The admission consent covers routine diagnostic procedures. 4. Pancreatic enzymes are administered when the pancreas cannot produce amylase and lipase, not when the beta cells cannot produce insulin.

The nurse administered 25 units of Humulin N to a client with Type 1 diabetes at 1600. Which intervention should the nurse implement? 1. Assess the client for hypoglycemia around 1800. 2. Ensure the client eats the nighttime snack. 3. Check the client's serum blood glucose level. 4. Serve the client the supper tray.

2 1. Humulin N is an intermediate-acting insulin that peaks 6-8 hours after adminis- tration; therefore, the client would experi- ence signs of hypoglycemia around 2200-2400. 2. The nurse needs to ensure the client eats the nighttime (HS) snack to help prevent nighttime hypoglycemia. 3. A serum blood glucose level would have to be done with a venipuncture and the blood sample must be taken to the laboratory. If the client needed the blood glucose checked, it should be done with a glucometer at the bedside. 4. The supper tray would not help prevent a hypoglycemic reaction because the Humulin N is an intermediate-acting insulin that peaks in 6-8 hours.

The client who has been using oxymetazoline (Afrin) nasal spray for several weeks complains to the nurse that the spray no longer seems to work to clear the nasal passages. Which information should the nurse teach? 1. Increase the amount of sprays used until the desired effect has been reached. 2. This type of medication can cause rebound congestion if used too long. 3. Alternate the Afrin with a saline nasal spray every 2 hours. 4. Place the Afrin nasal spray in a vaporizer at night for the best results.

2 1. Increasing the number of sprays will only increase the problem. This medication is for short-term use only (that is, a few days). Longer use can cause a rebound congestion that can be difficult to resolve. 2. Afrin is recommended for short-term relief of nasal congestion for clients older than the age of 6 years. Longer use can cause a rebound congestion that can be difficult to resolve. 3. Afrin should be used every 10-12 hours only; using it more often increases the chance of developing a dependence on the medication and rebound congestion. 4. Afrin nasal spray is to be used intranasally; it is not an additive for a vaporizer.

The client diagnosed with Type 1 diabetes is complaining of a dry mouth, extreme thirst, and increased urination. Which action should the nurse implement? 1. Administer one amp of intravenous 50% glucose. 2. Prepare to administer intravenous regular insulin. 3. Inject Humulin N subcutaneously in the abdomen. 4. Hang an intravenous infusion of D5W at a keep open rate.

2 1. One amp of 50% glucose would be used to treat a severe hypoglycemic reaction, and this client does not have signs or symptoms that indicate hypoglycemia. In fact, the client has signs and symptoms of hyper- glycemia. 2. The client's signs and symptoms indicate the client is experiencing diabetic ketoacidosis (DKA), which is treated with intravenous regular insulin. 3. Humulin N is an intermediate-acting insulin, which is not used to treat hyper- glycemia. 4. An IV of D5W would cause the client to have further signs and symptoms of diabetic ketoacidosis (DKA); therefore, the nurse should not administer the IV.

The UAP on the medical floor tells the nurse the client diagnosed with DKA wants something else to eat for lunch. Which intervention should the nurse implement? 1. Instruct the UAP to get the client additional food. 2. Notify the dietitian about the client's request. 3. Request the HCP increase the client's caloric intake. 4. Tell the UAP the client cannot have anything else.

2 1. The client is on a special diet and should not have any additional food. 2. The client will not be compliant with the diet if he or she is still hungry. Therefore, the nurse should request the dietitian talk to the client to try to adjust the meals so the client will adhere to the diet. 3. The nurse does not need to notify the HCP for an increase in caloric intake. The appropriate referral is to the dietitian. 4. The client is on a special diet. The nurse needs to help the client maintain compli- ance with the medical treatment and should refer the client to the dietitian.

The client is getting out of bed and becomes very anxious and has a feeling of impending doom. The nurse thinks the client may be experiencing a pulmonary embolism. Which action should the nurse implement first? 1. Administer oxygen 10 L via nasal cannula. 2. Place the client in high Fowler's position. 3. Obtain a STAT pulse oximeter reading. 4. Auscultate the client's lung sounds.

2 1. The client needs oxygen, but the nurse can intervene to help the client before applying oxygen. 2. Placing the client in this position facilitates maximal lung expansion and reduces venous return to the right side of the heart, thus lowering pressures in the pulmonary vascular system. 3. A pulse oximeter reading is needed, but it is not the first intervention. 4. Assessing the client is indicated, but it is not the first intervention in this situation.

The nurse is caring for the client diagnosed with Type 2 diabetes. The client is complaining of a headache, jitteriness, and nervousness. Which action should the nurse implement first? 1. Check the client's serum blood glucose level. 2. Give the client a glass of orange juice. 3. Determine when the last antidiabetic medication was administered. 4. Assess the client's blood pressure and apical pulse.

2 1. The client's serum blood glucose level is checked by drawing a venipuncture blood sample and sending it to the laboratory. This would take too long. The nurse must take care of the client; therefore, drawing a blood sample and awaiting results is not the first intervention. 2. The client is experiencing signs of a hypoglycemic reaction and the nurse must treat the client by administering some type of simple-acting glucose. This is the first intervention. 3. Determining when the last oral hypo- glycemic medication was administered is an intervention that could be imple- mented, but it is not the first intervention. The nurse needs to take care of the client. 4. The nurse could assess the client's vital signs, but this is not the first intervention; the nurse should take care of the client's signs and symptoms. MEDICATION MEMORY JOGGER: When answering test questions or when caring for clients at the bedside, the nurse should remember that assessing the client might not be the first action to take when the client is in distress. The nurse may need to intervene directly to help the client.

The client diagnosed with chronic obstructive pulmonary disease (COPD) is prescribed morphine sulfate (MS Contin). Which statement is the scientific rationale for prescribing this medication? 1. MS Contin will depress the respiratory drive. 2. Morphine dilates the bronchi and improves breathing. 3. MS Contin is not addicting, so it can be given routinely. 4. Morphine causes bronchoconstriction and decreased sputum.

2 1. The nurse does not administer medica- tions to decrease the respiratory drive for any client—especially not one diagnosed with pulmonary disease. 2. Morphine is a mild bronchodilator, and the continuous-release formulation provides a sustained effect for the client. 3. All forms of morphine can be addicting. 4. Bronchoconstriction would increase the client's difficulty in breathing and trap sputum below the constricted bronchus.

The client with chronic reactive airway disease is taking the leukotriene receptor inhibitor montelukast (Singulair). Which statement by the client would warrant intervention by the nurse? 1. "I have been having a lot of headaches lately." 2. "I have started taking an aspirin every day." 3. "I keep this medication up on a very high shelf." 4. "I must protect this medication from extreme temperatures."

2 1. These drugs are generally safe and well- tolerated, with a headache being the most common side effect; therefore, this state- ment would not warrant intervention by the nurse. 2. This medication interacts with aspirin, warfarin, erythromycin, and theophyl- line; therefore, this statement warrants further intervention by the nurse. 3. All medications should be kept out of the reach of children, and keeping the medica- tion on a high shelf would not warrant intervention by the nurse. 4. This medication does not need to be kept from extreme temperatures; it is the anti- asthmatic zafirlukast (Accolate) that must be protected from extremes of tempera- ture, light, and humidity. MEDICATION MEMORY JOGGER: If the client verbalizes a complaint, if the nurse assesses data, or if laboratory data indi- cates an adverse effect secondary to a medication, the nurse must intervene. The nurse must implement an independ- ent action during intervention or notify the health-care provider because medica- tions can result in serious or even life- threatening complications.

Which data would indicate that the antibiotic therapy has not been successful for a client diagnosed with a bacterial pneumonia? 1. The client's hematocrit is 45%. 2. The client is expectorating thick, green sputum. 3. The client's lung sounds are clear to auscultation. 4. The client has no complaints of pleuritic chest pain.

2 1. This hematocrit is normal, but this does not indicate that the client is responding to the antibiotics. 2. Thick, green sputum is a symptom of pneumonia, which indicates the antibi- otic therapy is not effective. If the sputum were changing from a thick, green sputum to a thinner, lighter- colored sputum, it would indicate an improvement in the condition. 3. The symptoms of pneumonia include crackles and wheezing in the lung fields. Clear lung sounds indicate an improve- ment in the pneumonia and that the medication is effective. 4. Pleuritic chest is a symptom of pneumo- nia, and no chest pain indicates the medication is effective. MEDICATION MEMORY JOGGER: The nurse determines the effectiveness of a medication by assessing for the symp- toms, or lack thereof, for which the medication was prescribed.

The nurse at a freestanding health care clinic is caring for a 56-year-old male client who is homeless and is a type 2 diabetic controlled with insulin. Which action is an example of client advocacy? 1. Ask the client if he has somewhere he can go and live. 2. Arrange for someone to give him insulin at a local homeless shelter. 3. Notify Adult Protective Services about the client's situation. 4. Ask the HCP to take the client off insulin because he is homeless.

2 1. This is an example of interviewing the client; it is not an example of client advocacy. 2. Client advocacy focuses support on the client's autonomy. Even if the nurse disagrees with his living on the street, it is the client's right. Arranging for someone to give him his insulin provides for his needs and allows his choices. 3. Adult Protective Services is an organiza- tion investigating any actual or potential abuse in adults. This client is not being abused by anyone. 4. The client needs the insulin to control the diabetes, and talking to the HCP about taking him off a needed medication is not an example of advocacy.

The nurse is preparing to administer medications on a pulmonary unit. Which medication should the nurse administer first? 1. Prednisone, a glucocorticoid, for a client diagnosed with chronic bronchitis. 2. Oxygen via nasal cannula at 2 L/min for a client diagnosed with pneumonia. 3. Lactic acidophilus (Lactinex) to a client receiving IVPB antibiotics. 4. Cephalexin (Keflex), an antibiotic, to a client being discharged.

2 1. This is an oral preparation and one that can be given daily; this is not the first medication to be administered. 2. Oxygen is considered a medication and should be a priority whenever it is ordered. A client diagnosed with pneu-monia will have some amount of respi- ratory compromise, and the ordered 2 L/min indicates a client with a chronic lung disease. This is the priority medication. 3. Lactinex is administered to replace the good bacteria in the body destroyed by the antibiotic, but it does not need to be administered first. 4. Keflex is an oral antibiotic, but this client is being discharged, indicating the client's condition has improved. This client could wait until the oxygen is initiated. MEDICATION MEMORY JOGGER: Oxygen is a medication, and the nurse should remember basic principles that apply to oxygen administration. The test taker could choose the correct answer based on Maslow's Hierarchy of Needs and breath- ing/oxygen is the priority.

The client diagnosed with a pulmonary embolus (PE) is receiving intravenous heparin, and the HCP prescribes 5 mg warfarin (Coumadin) orally once a day. Which statement best explains the scientific rationale for prescribing these two anticoagulants? 1. Coumadin interferes with production of prothrombin. 2. It takes 3-5 days to achieve a therapeutic level of Coumadin. 3. Heparin is more effective when administered with warfarin. 4. Coumadin potentiates the therapeutic action of heparin.

2 1. This is the scientific rationale for why Coumadin is prescribed to prevent throm- bus formation, but it is not the rationale for why the medications are administered together. 2. Heparin has a short half-life and is prescribed as soon as a PE is suspected. The client must go home having taken an oral anticoagulant such as Coumadin, which has a long half-life and needs at least 3-5 days to reach a therapeutic level. Discontinuing the heparin prior to achieving a therapeu- tic level of Coumadin places the client at risk for another PE. 3. Heparin and warfarin work in different steps in the bleeding cascade. 4. This is a false statement; heparin and warfarin work in different steps in the bleeding cascade.

An 18-year-old female client, 5′4′′ tall, weighing 113 kg, comes to the clinic for a nonhealing wound on her lower leg, which she has had for two (2) weeks. Which disease process should the nurse suspect the client has developed? 1. Type 1 diabetes. 2. Type 2 diabetes. 3. Gestational diabetes. 4. Acanthosis nigricans.

2 1. Type 1 diabetes usually occurs in young clients who are underweight. In this disease, there is no production of insulin from the beta cells in the pancreas. People with type 1 diabetes are insulin dependent with a rapid onset of symptoms, including polyuria, polydipsia, and polyphagia. 2. Type 2 diabetes is a disorder usually occurring around the age of 40, but it is now being detected in children and young adults as a result of obesity and sedentary lifestyles. Nonhealing wounds are a hallmark sign of type 2 diabetes. This client weighs 248.6 pounds and is short. 3. Gestational diabetes occurs during pregnancy. 4. Acanthosis nigricans (AN), dark pigmenta- tion and skin creases in the neck, is a sign of hyperinsulinemia. The pancreas is secreting excess amounts of insulin as a result of excessive caloric intake. It is identified in young children and is a precursor to the development of type 2 diabetes.

Which statement best describes the scientific rationale for prescribing the biguanide metformin (Glucophage)? 1. This medication decreases insulin resistance, improving blood glucose control. 2. This medication allows the carbohydrates to pass slowly through the large intestine. 3. This medication will decrease the hepatic production of glucose from stored glycogen. 4. This medication stimulates the beta cells to release more insulin into the blood- stream.

3 1. A thiazolidinedione, pioglitazone (Actos) or rosiglitazone (Avandia), not a biguanide like metformin, is prescribed to decrease insulin resistance. 2. An alpha-glucosidase inhibitor, acarbose (Precose) or miglitol (Glyset), is adminis- tered to allow carbohydrates to pass slowly through the intestine. Glucophage does not do this. 3. The scientific rationale for administer- ing metformin (Glucophage) is that it diminishes the increase in serum glucose following a meal and blunts the degree of postprandial hyperglycemia by preventing gluconeogenesis. 4. A meglitinide, repaglinide (Prandin), sulfonylurea, or nateglinide (Starlix) is prescribed to stimulate the beta cells to release more insulin into the bloodstream.

The client diagnosed with Type 2 diabetes is receiving the combination oral antidia- betic medication glyburide/metformin (Glucovance). Which data indicate the medication is effective? 1. The client's skin turgor is elastic. 2. The client's urine ketones are negative. 3. The serum blood glucose level is 118 mg/dL. 4. The client's glucometer level is 170 mg/dL.

3 1. An elastic skin turgor is expected and normal, but it does not indicate that the antidiabetic medication is effective. 2. Urine ketones should be negative because there should not be a breakdown of fat in clients with Type 2 diabetes, but this does not indicate the effectiveness of the medication. 3. The serum blood glucose level should be within normal limits, which is 70-110 mg/dL. A level of 118 mg/dL is close to normal; therefore, the medica- tion can be considered effective. 4. A self-monitoring blood glucose level of 170 mg/dL is above a normal glucose level; this indicates the medication is not effective. MEDICATION MEMORY JOGGER: The nurse determines the effectiveness of a medication by assessing for the symp- toms, or lack thereof, for which the medication was prescribed.

The client diagnosed with rule-out deep vein thrombosis (DVT) is experiencing dyspnea and chest pain on inspiration. On assessment, the nurse finds a respiratory rate of 40. Which medication should the nurse anticipate the health-care provider ordering? 1. Warfarin (Coumadin), an oral anticoagulant. 2. Enoxaparin (Lovenox), a low molecular weight heparin. 3. Heparin, an intravenous anticoagulant. 4. Ticlopidine (Ticlid), an antiplatelet medication.

3 1. An oral anticoagulant would not be prescribed in an acute situation. 2. Lovenox is prescribed prophylactically to prevent deep vein thrombosis. The client is currently experiencing a complication of DVT; therefore, the nurse should not anticipate an order for this medication. 3. Heparin is the medication of choice for treating a pulmonary embolus, which the nurse should suspect with these signs and symptoms. Intravenous heparin will prevent further clotting. 4. Ticlid is a medication used to treat arte- rial, not venous, conditions. MEDICATION MEMORY JOGGER: Remember that antiplatelets work in the arteries and anticoagulants work in the veins.

The client diagnosed with emphysema is admitted to the surgical unit for a cholecys- tectomy (gallbladder removal). Which postoperative intervention should the nurse implement? 1. Have the patient turn, cough, and breathe deeply every shift. 2. Administer oxygen to the client at 4 L/min. 3. Assess the surgical site for delayed healing. 4. Medicate frequently with morphine 15 mg IVP.

3 1. Clients undergoing surgery are encour- aged to turn, cough, and deep breathe (TC&DB) a minimum of every 2 hours. Clients with emphysema should TC&DB more often than every 2 hours. 2. The client should be administered oxygen at 1-3 L/min. Clients with chronic lung disease have developed carbon dioxide narcosis; high levels of carbon dioxide have destroyed the client's first stimulus for breathing. Oxygen hunger is the body's backup system for sustaining life. Administering oxygen at levels above 2 L/min at rest and 3 L/min during activity may cause the client to stop breathing. 3. Clients diagnosed with chronic lung disease are frequently prescribed long- term steroid therapy. Steroids delay wound healing. The nurse should assess the wound to determine that the surgical incision is healing as desired. 4. Morphine can cause respiratory compro- mise, especially when given frequently and in large doses. This client is already at risk for respiratory complications from the emphysema.

Which over-the-counter herb should the nurse recommend for a client with a cold who has mild hypertension? 1. Crataegus laevigata, hawthorn. 2. Zingiber officinale, ginger. 3. Allium sativum, garlic. 4. Hydrastis canadensis, goldenseal.

3 1. Hawthorn is used for mild hypertension, congestive heart failure, and angina, but it does nothing for a cold or the flu. 2. Ginger is used to stimulate digestion and to help ease nausea and motion sickness. It does nothing for hypertension or the flu or colds. 3. Garlic is used for colds and the flu and can also be given for hypertension. It causes mild vasodilation and will not make hypertension worse. 4. Goldenseal is used for respiratory, digestive, and urinary infections, but it increases the effectiveness of some antihypertensive medications, beta blockers, and antidys- rhythmics. It should be used with caution in clients who have cardiovascular disease, diabetes mellitus, or glaucoma. MEDICATION MEMORY JOGGER: Some herbal preparations are effective, some are not, and a few can be harmful or even deadly. If a client is taking an herbal supplement and a conventional medicine, the nurse should investigate to determine if the combination will cause harm to the client. The nurse should always be the client's advocate.

The nurse is teaching a client with newly diagnosed Type 1 diabetes about insulin therapy. Which statement indicates the client needs more teaching concerning insulin therapy? 1. "If I have a headache or start getting nervous, I will drink some orange juice." 2. "If I pass out at home, a family member should give me a glucagon injection." 3. "Because I am taking my insulin daily I do not have to adhere to a diabetic diet." 4. "I will check my blood glucose with my glucometer at least once a day."

3 1. Headache, nervousness, sweating, tremors, and rapid pulse are signs of a hypoglycemic reaction and should be treated with a simple-acting carbohydrate, such as orange juice, sugar-containing drinks, and hard candy. This statement indicates the client understands the teaching. 2. If a client cannot drink or eat a simple carbohydrate for hypoglycemia, then the client should receive a glucagon injection to treat the hypoglycemic reaction. This indi- cates the client understands the teaching. 3. Even with insulin therapy the client should adhere to the American Diabetic Association diet, which recommends "carbohydrate counting." This state- ment indicates the client needs more teaching. 4. Monitoring and documenting the blood glucose level is encouraged to determine the effectiveness of the treatment regimen. This indicates the client understands the client teaching.

The client diagnosed with type 2 diabetes is admitted to the intensive care unit with hyperosmolar hyperglycemic nonketonic syndrome (HHNS) coma. Which assessment data should the nurse expect the client to exhibit? 1. Kussmaul's respirations. 2. Diarrhea and epigastric pain. 3. Dry mucous membranes. 4. Ketone breath odor.

3 1. Kussmaul's respirations occur with diabetic ketoacidosis (DKA) as a result of the breakdown of fat, resulting in ketones. 2. Diarrhea and epigastric pain are not associated with HHNS. 3. Dry mucous membranes are a result of the hyperglycemia and occur with both HHNS and DKA. 4. This occurs with DKA as a result of the breakdown of fat, resulting in ketones.

The nurse is discussing ways to prevent diabetic ketoacidosis with the client diagnosed with type 1 diabetes. Which instruction is most important to discuss with the client? 1. Refer the client to the American Diabetes Association. 2. Do not take any over-the-counter medications. 3. Take the prescribed insulin even when unable to eat because of illness. 4. Explain the need to get the annual flu and pneumonia vaccines.

3 1. The American Diabetes Association is an excellent referral, but the nurse should discuss specific ways to prevent DKA. 2. The client should be careful with OTC medications, but this intervention does not help prevent the development of DKA. 3. Illness increases blood glucose levels; therefore, the client must take insulin and consume high-carbohydrate foods such as regular Jell-O, regular popsi- cles, and orange juice. 4. Vaccines are important to help prevent ill- ness, but regardless of whether the client gets these vaccines, the client can still de- velop diabetic ketoacidosis.

The nurse is administering alteplase (Activase), a thrombolytic, to a client diagnosed with massive pulmonary emboli (PE). Which data indicates the medication is effective? 1. The client's PTT level is within therapeutic range. 2. The client is able to ambulate to the bathroom. 3. The client denies chest pain on inspiration. 4. The client's chest x-ray is normal.

3 1. The PTT test is used to monitor the anti- coagulant heparin, not the thrombolytic Activase. 2. A client with a massive PE would be on bed rest; therefore, ambulating would not indicate the medication is effective. 3. To determine if the medication is effective, the nurse must assess for an improvement in the signs or symptoms for the condition for which the medica- tion was ordered. Chest pain is one of the most common symptoms of PE; denial of chest pain would indicate the medication is effective. 4. In the client diagnosed with a PE the chest x-ray is usually normal; therefore, it would not be used to determine if the thrombolytic is effective.

The client diagnosed with type 1 diabetes has a glycosylated hemoglobin (A1c) of 8.1%. Which interpretation should the nurse make based on this result? 1. This result is below normal levels. 2. This result is within acceptable levels. 3. This result is above recommended levels. 4. This result is dangerously high.

3 1. The acceptable level for an A1c for a client with diabetes is between 6% and 7%, which corresponds to a 120- to 140-mg/dL aver- age blood glucose level. 2. This result is not within acceptable levels for the client with diabetes, which is 6% to 7%. 3. This result parallels a serum blood glucose level of approximately 180 to 200 mg/dL. An A1c is a blood test reflect- ing average blood glucose levels over a period of three (3) months; clients with elevated blood glucose levels are at risk for developing long-term complications. 4. An A1c of 13% is dangerously high; it re- flects a 300-mg/dL average blood glucose level over the past 3 months.

The client received 10 units of Humulin R, a fast-acting insulin, at 0700. At 1030 the unlicensed assistive personnel (UAP) tells the nurse the client has a headache and is really acting "funny." Which intervention should the nurse implement first? 1. Instruct the UAP to obtain the blood glucose level. 2. Have the client drink eight (8) ounces of orange juice. 3. Go to the client's room and assess the client for hypoglycemia. 4. Prepare to administer one (1) ampule 50% dextrose intravenously.

3 1. The blood glucose level should be ob- tained, but it is not the first intervention. 2. If it is determined the client is having a hypoglycemic reaction, orange juice is appropriate. 3. Regular insulin peaks in 2 to 4 hours. Therefore, the nurse should think about the possibility the client is hav- ing a hypoglycemic reaction and should assess the client. The nurse should not delegate nursing tasks to a UAP if the client is unstable. 4. Dextrose 50% is only administered if the client is unconscious and the nurse sus- pects hypoglycemia.

The client is prescribed albuterol (Ventolin), a sympathomimetic bronchodilator, metered-dose inhaler. Which behavior indicates the teaching concerning the inhaler is effective? 1. The client holds his or her breath for 5 seconds and then exhales forcefully. 2. The client states the canister is full when it is lying on top of the water. 3. The client exhales and then squeezes the canister as the next inspiration occurs. 4. The client connects the oxygen tubing to the inhaler before administering the dose.

3 1. The client should hold his or her breath as long as possible before exhaling to allow the medication to settle beforeadministering another dose; 5 seconds is not long enough. 2. The client can check how much medica- tion is in a metered-dose canister by plac- ing the canister in a glass of water; if the canister stays under water, the canister is full, and if it floats on top of the water, it is empty. 3. This is the correct way to use an inhaler because it will carry the medication down into the lung. 4. Oxygen is not used when using an inhaler; oxygen is used to deliver the medication when using an aerosol.

Which information should the nurse discuss with the client diagnosed with reactive airway disease who is prescribed theophylline (Slo-Phyllin), a xanthine bronchodilator? 1. Instruct the client to take the medication on an empty stomach. 2. Explain that an increased heart rate and irritability are expected side effects. 3. Discuss the need to avoid large amounts of caffeine-containing drinks. 4. Tell the client to double the next dose if a dose is missed.

3 1. The client should take the medication with a glass of water or with meals to avoid an upset stomach. 2. The client should notify the health-care provider of a rapid or irregular heartbeat, vomiting, dizziness, or irritability because these are not expected side effects. 3. The client should avoid drinking large amounts of caffeine-containing drinks such as tea, coffee, cocoa, and cola drinks. 4. If a dose is missed within an hour, the client should take the dose immediately, but if it is more than 1 hour, the client should skip the dose and stay on the origi- nal dosing schedule. The client should not double the dose.

The school nurse is teaching a class about Type 2 diabetes to elementary school teach- ers. Which information is most important for the nurse to discuss with the teachers? 1. The importance of not allowing students to eat candy in the classroom. 2. The increase in the number of students developing Type 2 diabetes. 3. The signs and symptoms of hypoglycemia and the immediate treatment. 4. The need to have the students run or walk for 20 minutes during the recess period.

3 1. The students with Type 2 diabetes should not eat candy, but it is not the most important intervention for the school nurse to teach. 2. This is pertinent information, but it is not the most important information. 3. The most important information for the teachers to know is how to treat potentially life-threatening complica- tions secondary to the medications used to treat Type 2 diabetes. The school nurse should discuss issues that keep the students safe. 4. Exercise is important in helping to control Type 2 diabetes, but empowering the teachers to be confident when handling complications secondary to medication is priority for the safety of the students.

The client admitted for an acute exacerbation of reactive airway disease is receiving intravenous aminophylline. The client's serum theophylline level is 28 g/mL. Which action should the nurse implement first? 1. Continue to monitor the aminophylline drip. 2. Assess the client for nausea and restlessness. 3. Discontinue the aminophylline drip. 4. Notify the health-care provider immediately.

3 1. The therapeutic level for theophylline is 10-20 g/mL; therefore, the nurse should take action. 2. As the serum theophylline level rises above 20 g/mL, the client will experi- ence nausea, vomiting, diarrhea, insomnia, and restlessness. This theophylline level may result in serious effects, such as convulsion and ventricular fibrillation. Therefore, the client should not be assessed first. 3. The client has the potential for having convulsions and ventricular fibrillation because the theophylline level is too high; therefore, the nurse should discontinue the aminophylline drip first. 4. After discontinuing the aminophylline drip and then assessing the client for potential life-threatening complications, the nurse should notify the health-care provider. MEDICATION MEMORY JOGGER: The nurse must be knowledgeable about accepted standards of practice for medica- tion administration, including which client assessment data and laboratory data should be monitored prior to administer- ing the medication.

The female client asks the nurse why her teenage child would have many boxes of Sudafed, an over-the-counter cold and allergy medication, in her room. Which would be the nurse's first response? 1. "Has your child always had allergy problems?" 2. "Teenagers will try to take care of their own health problems." 3. "Has the teenager's behavior at school or at home changed recently?" 4. "Remove the medication and say nothing to the teenager about it."

3 1. These may be allergy medications when used legally, but they are also the ingredi-ents in illegal methamphetamine produc- tion. Quantities of any medication in a teenager's room should be investigated. 2. Teenagers do try to develop independence, but it is always the parent or guardian's responsibility to monitor the child's health. 3. This situation could indicate the teen- ager is involved with the drug culture, taking or manufacturing drugs. The nurse should assess for signs of drug involvement. 4. The parent is responsible for determining the teenager's activities; the situation should be discussed with the teenager.

The charge nurse is making client assignments in the intensive care unit. Which client should be assigned to the most experienced nurse? 1. The client with type 2 diabetes who has a blood glucose level of 348 mg/dL. 2. The client diagnosed with type 1 diabetes who is experiencing hypoglycemia. 3. The client with DKA who has multifocal premature ventricular contractions. 4. The client with HHNS who has a plasma osmolarity of 290 mOsm/L.

3 1. This blood glucose level is elevated, but not life threatening, in the client diagnosed with type 2 diabetes. Therefore, a less ex- perienced nurse could care for this client. 2. Hypoglycemia is an acute complication of type 1 diabetes, but it can be managed by frequent monitoring, so a less experienced nurse could care for this client. 3. Multifocal PVCs, which are secondary to hypokalemia and can occur in clients with DKA, are a potentially life-threatening emergency. This client needs an experienced nurse. 4. A plasma osmolarity of 280 to 300 mOsm/L is within normal limits; therefore, a less experienced nurse could care for this client.

The client with an acute exacerbation of reactive airway disease is prescribed a nebu- lizer treatment. Which statement best describes how a nebulizer works? 1. Nebulizers are small, handheld pressurized devices that deliver a measured dose of an antiasthma drug with activation. 2. A nebulizer is an inhaler that delivers an antiasthma drug in the form of a dry, micronized power directly to the lungs. 3. A nebulizer is a small machine used to convert an antiasthma drug solution into a mist that is delivered though a mouthpiece. 4. Nebulizers are small devices that are used to crush glucocorticoids so that the client can place them under the tongue for better absorption.

3 1. This is the description of how a metered- dose inhaler works. 2. This is the description of how a dry- powder inhaler works. 3. This is the description of how a nebu- lizer works. Nebulizers take several minutes to deliver the same amount of drug contained in one puff from an inhaler. They are usually used at home but can be used in the hospital. 4. This is not the description of how a nebu- lizer works. Glucocorticoids are not used sublingually to treat acute or chronic asthma.

The client diagnosed with chronic obstructive pulmonary disease is prescribed methylprednisolone (Solu-Medrol), a glucocorticoid, IVP. Which laboratory test should the nurse monitor? 1. The white blood cell (WBC) count. 2. The hemoglobin and hematocrit. 3. The blood glucose level. 4. The BUN and creatinine.

3 1. White blood cells are monitored to detect the presence of an infection, not for steroids. 2. The hemoglobin and hematocrit are monitored to detect blood loss, not for steroid therapy. 3. Steroid therapy interferes with glucose metabolism and increases insulin resistance. The blood glucose levels should be monitored to determine if an intervention is needed. 4. The BUN and creatinine levels are moni- tored to determine renal status. The adre- nal glands produce cortisol. MEDICATION MEMORY JOGGER: The nurse must be knowledgeable about accepted standards of practice for medica- tion administration including which client assessment data and laboratory data should be monitored prior to administer- ing the medication.

The client diagnosed with Type 2 diabetes is prescribed the sulfonylurea glipizide (Glucotrol). Which statement by the client would warrant intervention by the nurse? 1. "I have to eat my diabetic diet even if I am taking this medication." 2. "I will need to check my blood glucose level at least once a day." 3. "I usually have one glass of wine with my evening meal." 4. "I do not like to walk every day, but I will if it will help my diabetes."

3 11. 1. The client with Type 2 diabetes must adhere to the prescribed diet to help keep the blood glucose level within the normal range. Delaying or missing a meal can cause hypoglycemia. This state- ment would not warrant intervention by the nurse. 2. The client should check blood glucose levels to determine if the medication is effective; therefore, this statement would not warrant intervention by the nurse. 3. Sulfonylureas and biguanides may cause an Antabuse-like reaction when taken with alcohol, causing the client to become nauseated and vomit. Advise the client to abstain from alcohol and to avoid liquid over-the-counter (OTC) medications that may contain alcohol. Alcohol also increases the half-life of the medication and can cause a hypo- glycemic reaction. 4. The client with Type 2 diabetes does not need to walk daily to keep the glucose level within normal limits; walking three times a week will help control stress and help decrease weight if the client is over- weight.

The nurse in the medical department is preparing to administer Humalog, a rapid- acting insulin, to a client diagnosed with Type 1 diabetes. Which intervention should the nurse implement? 1. Ensure the client is wearing a MedicAlert bracelet. 2. Administer the dose according to the regular insulin sliding scale. 3. Assess the client for hyperosmolar, hyperglycemic, nonketotic coma. 4. Make sure the client eats the food on the meal tray that is at the bedside.

4 1. Because the client is in the hospital the client must have a hospital identification band; a MedicAlert bracelet would be needed when the client is not in the hospital. 2. Humalog is not regular insulin; it is fast- acting insulin. It is not administered accord- ing to the regular insulin sliding scale. The peak time for Humalog is 30 minutes to 1 hour; regular insulin peaks in 2-4 hours. 3. A client with Type 1 diabetes will experi- ence diabetic ketoacidosis; a client with Type 2 diabetes will experience hyperosmo- lar, hyperglycemic, nonketotic coma. 4. Humalog peaks in 30 minutes to 1 hour; therefore, the client needs to eat when or shortly after the medication is admin- istered to prevent hypoglycemia. MEDICATION MEMORY JOGGER: Remember that the different types of insulin peak at different times, and the nurse must be knowledgeable about the peak times to ensure that the client does not experience hypoglycemia. Only the insulin product Lantus has no peak time.

The male client diagnosed with chronic obstructive pulmonary disease (COPD) tells the nurse that he has been expectorating "rusty-colored" sputum. Which medication would the nurse anticipate the HCP prescribing? 1. Prednisone, a glucocorticoid. 2. Habitrol, a transdermal nicotine system. 3. Dextromethorphan (Robitussin), an antitussive. 4. Ceftriaxone (Rocephin), a cephalosporin.

4 1. Clients diagnosed with COPD are commonly prescribed a steroid (glucocor- ticoid) medication to decrease inflamma- tion in the lungs. This client should already be taking this or a similar medica- tion. The client's "rusty-colored" sputum indicates an infection and an antibiotic should be ordered. 2. The client should quit smoking if still smoking, but the client's "rusty-colored" sputum indicates an infection and an antibiotic should be ordered. 3. The client may require an antitussive but more likely would require a mucolytic to help to expectorate the thick tenacious sputum associated with COPD. 4. The client's "rusty-colored" sputum indicates an infection and an antibiotic should be ordered. Rocephin is a broad-spectrum antibiotic.

The client with Type 2 diabetes is admitted into the medical department with a wound on the left leg that will not heal. The HCP prescribes sliding-scale insulin. The client tells the nurse, "I don't want to have to take shots. I take pills at home." Which statement would be the nurse's best response? 1. "If you can't keep your glucose under control with pills, you must take insulin." 2. "You should discuss the insulin order with your HCP because you don't want to take it." 3. "You are worried about having to take insulin. I will sit down and we can talk." 4. "During illness you may need to take insulin to keep your blood glucose level down."

4 1. During illness, the client with Type 2 diabetes may need insulin to help keep glucose levels under control, but this is a threatening type of statement and is not the nurse's best response. 2. Insulin may need to be prescribed in times of stress, surgery, or serious infection; therefore, the nurse should explain this to the client and not refer the client to the HCP. 3. This is a therapeutic response and the client needs to have factual information. Therapeutic responses are used to encour- age the client to ventilate feelings. 4. Blood glucose levels elevate during times of stress, surgery, or serious infection. The client with Type 2 diabetes may need to be given insulin temporarily to help keep the blood glucose level with normal limits.

The client newly diagnosed with Type 2 diabetes who has been prescribed an oral hypoglycemic medication calls the clinic and tells the nurse that the sclera has a yellow color. Which action should the clinic nurse implement? 1. Ask the client if he or she has been exposed to someone with hepatitis. 2. Determine if the client has a history of alcohol use or is currently drinking alcohol. 3. Check to see if the client is taking the cardiac glycoside digoxin. 4. Make an appointment for the client to come to the health-care provider's office.

4 1. Jaundiced sclera may indicate the client has hepatitis, but because the client has been prescribed oral hypoglycemic medications, their possible role in the development of the jaundice should be assessed. 2. The nurse should not jump to the conclu- sion that the client is an alcoholic just because the sclera is jaundiced. 3. Digoxin toxicity results in the client having a yellow haze, not the client's sclera being yellow. 4. Oral hypoglycemics are metabolized in the liver and may cause elevations in liver enzymes; the client should be instructed to report the first signs of yellow skin, sclera, pale stools, or dark urine to the HCP.

The nurse is discussing the importance of exercising with a client diagnosed with type 2 diabetes whose diabetes is well controlled with diet and exercise. Which information should the nurse include in the teaching about diabetes? 1. Eat a simple carbohydrate snack before exercising. 2. Carry peanut butter crackers when exercising. 3. Encourage the client to walk 20 minutes three (3) times a week. 4. Perform warmup and cool-down exercises.

4 1. The client diagnosed with type 2 diabetes who is not taking insulin or oral agents does not need extra food before exercise. 2. The client with diabetes who is at risk for hypoglycemia when exercising should carrya simple carbohydrate, but this client is not at risk for hypoglycemia. 3. Clients with diabetes controlled by diet and exercise must exercise daily at the same time and in the same amount to control the glucose level. 4. All clients who exercise should perform warmup and cool-down exercises to help prevent muscle strain and injury.

The nurse is discharging the female client diagnosed with a pulmonary embolus (PE) who is prescribed the anticoagulant warfarin (Coumadin). Which statement indicates the client understands the medication teaching? 1. "I should use a straight razor when I shave my legs." 2. "I will use a hard-bristled toothbrush to clean my teeth." 3. "An occasional nosebleed is common with this drug." 4. "It will be important for me to have regular bloodwork done."

4 1. The client is at risk for bleeding and should be encouraged to use an electric razor. 2. The client is at risk for bleeding, and a soft-bristled toothbrush should be used. 3. Any abnormal bleeding, such as a nose- bleed, is not expected and should be reported to the HCP. Unexplained bleed- ing is a sign of toxicity. 4. The client's International Normalized Ratio (INR) is monitored at routine intervals to determine if the medica- tion is within the therapeutic range, INR 2-3.

The client diagnosed with arterial hypertension develops a cold. Which information regarding over-the-counter medications should the nurse teach? 1. Try to find a medication that will not cause drowsiness. 2. Over-the-counter medications are not as effective as a prescription. 3. Over-the-counter medications are more expensive than prescriptions. 4. Do not take over-the-counter medication unless approved by the HCP.

4 1. The client should be informed about the dangers of self-medicating with over-the- counter (OTC) medications. Many OTC medications work by causing vasoconstric- tion, which will increase the client's hyper- tension. 2. Efficacy of medications depends on the medication and strength. Most OTC medications were at one time prescription medications. There are many variables, and this statement is too general to be true. 3. The expense of the medications is not the relevant point for this client. The problem is to inform the client about the actions of many OTC medications and the effect on the client's hypertension. 4. Many OTC medications work by causing vasoconstriction, which will increase the client's hypertension; the client should only take medications (approved by the HCP) that will not affect the client's hypertension.

The female client diagnosed with Type 2 diabetes tells the clinic nurse that she started taking ginseng to help increase her memory. Which action should the clinic nurse take? 1. Take no action because ginseng does not affect Type 2 diabetes. 2. Determine what type of memory deficits the client is experiencing. 3. Explain that herbs are dangerous and she should not be taking them. 4. Determine if the client is currently taking any type of antidiabetic medication.

4 1. The nurse should investigate any herb the client is taking because most herbs do affect a disease process or the medication being taken for the disease process. 2. The nurse should determine if ginseng affects the client's Type 2 diabetes or medications that the client is taking for the disease process. 3. This is a negative, judgmental statement. Many herbs are beneficial to the client. The nurse should always assess the client and determine if the herb is detrimental to the client's disease process or affects the client's routine medication regimen prior to making this type of statement. 4. The nurse should determine if the client is taking any medication because many oral hypoglycemics interact with herbs. Ginseng and garlic may increase the hypoglycemic effects of oral hypo- glycemics. MEDICATION MEMORY JOGGER: Some herbal preparations are effective, some are not, and a few can be harmful or even deadly. If a client is taking an herbal supplement and a conventional medicine, the nurse should investigate to determine if the herbal preparation would cause harm to the client. The nurse should always be the client's advocate

Which statement best describes the pharmacodynamics of insulin? 1. Insulin causes the pancreas to secrete glucose into the bloodstream. 2. Insulin is metabolized by the liver and muscle and excreted in the urine. 3. Insulin is needed to maintain colloidal osmotic pressure in the bloodstream. 4. Insulin lowers blood glucose by promoting use of glucose in the body cells.

4 1. The pancreas does not secrete glucose. It secretes insulin, which is the key that opens the door to allow glucose to enter the body cells. Glucose enters the body through the gastrointestinal system. 2. This statement explains the pharmacoki- netics of insulin and how the body metab- olizes and excretes urine. Pharmacokinetics is the process of drug movement to achieve drug interaction. 3. Insulin does not maintain colloidal osmotic pressure. Albumin, a product of protein, maintains colloidal osmotic pressure. 4. This is the statement that explains the pharmacodynamics, which is the drug's mechanism of action or way that insulin is utilized by the body. Over time, elevated glucose levels in the bloodstream can cause long-term complications, including nephropathy, retinopathy, and neuropathy. Insulin lowers blood glucose by promoting the use of glucose in body cells.

The client diagnosed with HHNS was admitted yesterday with a blood glucose level of 780 mg/dL. The client's blood glucose level is now 300 mg/dL. Which intervention should the nurse implement? 1. Increase the regular insulin IV drip. 2. Check the client's urine for ketones. 3. Provide the client with a therapeutic diabetic meal. 4. Notify the HCP to obtain an order to decrease insulin.

4 1. The regular intravenous insulin is contin- ued because ketosis is not present, as with DKA. 2. The client diagnosed with type 2 diabetes does not excrete ketones in HHNS because there is enough insulin to prevent fat breakdown but not enough to lower blood glucose. 3. The client may or may not feel like eating, but it is not the appropriate intervention when the blood glucose level is reduced to 300 mg/dL. 4. When the glucose level is decreased to around 300 mg/dL, the regular insulin infusion therapy is decreased. Subcuta- neous insulin will be administered per sliding scale.

Which arterial blood gas results should the nurse expect in the client diagnosed with diabetic ketoacidosis? 1. pH 7.34, PaO2 99, PaCO2 48, HCO3 24. 2. pH 7.38, PaO2 95, PaCO2 40, HCO3 22. 3. pH 7.46, PaO2 85, PaCO2 30, HCO3 26. 4. pH 7.30, PaO2 90, PaCO2 30, HCO3 18.

4 1. This ABG indicates respiratory acidosis, which is not expected. 2. This ABG is normal, which is not expected. 3. This ABG indicates respiratory alkalosis, which is not expected. 4. This ABG indicates metabolic acidosis, which is expected in a client diagnosed with diabetic ketoacidosis.

The nurse is discharging a client diagnosed with chronic obstructive pulmonary disease (COPD). Which discharge instructions should the nurse provide regarding the client's prescription for prednisone, a glucocorticoid? 1. Take all the prednisone as ordered until the prescription is empty. 2. Take the prednisone on an empty stomach with a full glass of water. 3. Stop taking the prednisone if a noticeable weight gain occurs. 4. The medication should never be abruptly discontinued.

4 1. This is instruction for an antibiotic. Prednisone is not abruptly discontinued because cortisol (a glucocorticoid) is necessary to sustain life and the adrenal glands will stop producing cortisol while the client is taking it exogenously. 2. Prednisone can produce gastric distress; it is given with food to minimize the gastric discomfort. 3. Weight gain is a side effect of steroid ther- apy, and the client should not stop taking the medication if this occurs. This medication must be tapered off if the client is to stop the medication—if the client is able to discontinue the medica- tion at all. 4. Prednisone is not abruptly discontin- ued because cortisol (a glucocorticoid) is necessary to sustain life and the adre- nal glands stop producing cortisol while the client is taking it exogenously. The medication must be tapered off to prevent a life-threatening complication.

The 28-year-old female client with chronic reactive airway disease is taking the leukotriene receptor inhibitor montelukast sodium (Singulair). Which statement by the client indicates the client teaching is effective? 1. "I will not drink coffee, tea, or any type of cola drinks." 2. "I will take this medication at the beginning of an asthma attack." 3. "It is all right to take this medication if I am trying to get pregnant." 4. "I should not decrease the dose or suddenly stop taking this medication."

4 1. This medication does not stimulate the central nervous system; therefore, the client does not need to avoid caffeine- containing products. This statement indi- cates that the teaching is not effective. 2. These medications are not used to treat an acute exacerbation of reactive airway disease. They are adjunctive drugs given as part of the asthma regimen. This state- ment indicates the teaching is not effec- tive. 3. The safety of these drugs has not been established in pregnancy and breastfeed- ing. This statement indicates that the teaching has not been effective. 4. The client should not suddenly stop taking the medication or decrease the dose. This statement indicates the teaching has been effective. Singulair is used with other types of asthma medications and should be continued if the client has an acute asthma attack.

Which medical treatment is recommended for the client who is diagnosed with mild intermittent asthma? 1. This classification of asthma requires a combination of long-term control medica- tion plus a quick-relief medication. 2. Mild intermittent asthma needs a routine glucocorticoid inhaler and a sustained- relief theophylline. 3. This classification requires daily inhalation of an oral glucocorticoid and daily nebulizer treatments. 4. Mild intermittent asthma is treated on a PRN basis and no long-term control medication is needed.

4 1. This type of medical treatment would be used for a client with mild persistent asthma. 2. This medical treatment would be prescribed for a client with moderate persistent asthma. 3. The most severe class, severe persistent asthma, is managed with daily inhalation of a glucocorticoid (high dose), plus salmeterol, a long-acting inhaled agent. 4. Mild intermittent asthma is treated on a PRN basis; long-term control medication is not needed. The occa- sional acute attack is managed by inhaling a short-acting beta2 agonist. If the client needs the beta2 agonist more than twice a week, moving to Step 2 (mild persistent asthma) may be indicated.

Which is the scientific theory for prescribing zinc preparations for a client with a cold? 1. Zinc binds with the viral particle and reduces the symptoms of a cold. 2. Zinc decreases the immune system's response to a virus. 3. Zinc activates viral receptors in the body's immune system. 4. Zinc blocks the virus from binding to the epithelial cells of the nose.

4 1. Zinc does not bind the viral particle. Symptoms are diminished by blocking the ability of the virus to bind with the nasal lining. 2. Zinc is a micronutrient found in the body that helps to increase the body's immune system. 3. Activating viral receptors would increase the symptoms of a cold. 4. Theoretically, zinc blocks viral binding to nasal epithelium. Observation has shown that increased amounts of zinc can prevent the binding and prevent the development of symptoms of the rhinovirus.

The nurse is caring for a client diagnosed with a cold. Which is an example of an alternative therapy? 1. Vitamin C, 2,000 mg daily. 2. Strict bedrest. 3. Humidification of the air. 4. Decongestant therapy.

1 1. Alternative therapies are therapies not accepted as standard medical practice. These may be encouraged as long as they do not interfere with the medical regimen. Vitamin C in large doses is thought to improve the immune system's functions. 2. Bedrest is accepted standard advice for a client with a cold. 3. Humidifying the air helps to relieve congestion and is a standard practice. 4. Decongestant therapy is standard therapy for a cold.

The nurse is planning the care of a client diagnosed with asthma and has written a problem of "anxiety." Which nursing intervention should be implemented? 1. Remain with the client. 2. Notify the health-care provider. 3. Administer an anxiolytic medication. 4. Encourage the client to drink fluids.

1 1. Anxiety is an expected sequela of being unable to meet the oxygen needs of the body. Staying with the client lets the client know the nurse will intervene and the client is not alone. 2. Because anxiety is an expected occurrence with asthma, it is not necessary to notify the health-care provider. 3. An anxiolytic medication could decrease respiratory drive and increase the respiratory distress. Also, the medication will require a delayed time period to begin to work. 4. Drinking fluids will not treat an asthma attack or anxiety.

Which statement made by the client indicates the nurse's discharge teaching is effective for the client diagnosed with COPD? 1. "I need to get an influenza vaccine each year, even when there is a shortage." 2. "I need to get a vaccine for pneumonia each year with my influenza shot." 3. "If I reduce my cigarettes to six (6) a day, I won't have difficulty breathing." 4. "I need to restrict my drinking liquids to keep from having so much phlegm."

1 1. Clients diagnosed with COPD should receive the influenza vaccine each year. If there is a shortage, these clients have top priority. 2. The pneumococcal vaccine should be ad- ministered every five (5) to seven (7) years. 3. Reducing the number of cigarettes smoked does not stop the progression of COPD, and the client will continue to experience signs and symptoms such as shortness of breath or dyspnea on exertion. 4. Clients diagnosed with COPD should increase their fluid intake unless contraindicated for another health condition. The increased fluid assists the client in expectorating the thick sputum.

The nurse observes the unlicensed assistive personnel (UAP) entering an airborne isolation room and leaving the door open. Which action is the nurse's best response? 1. Close the door and discuss the UAP's action after coming out of the room. 2. Make the UAP come back outside the room and then reenter, closing the door. 3. Say nothing to the UAP but report the incident to the nursing supervisor. 4. Enter the client's room and discuss the matter with the UAP immediately.

1 1. Closing the door reestablishes the negative air pressure, which prevents the air from entering the hall and contaminating the hospital environment. When correcting an individual, it is always best to do so in a private manner. 2. The employee is an adult and as such should be treated with respect and corrected accordingly. 3. Problems should be taken care of at the lowest level possible. The nurse is responsible for any task delegated, including the appropriate handling of isolation. 4. Correcting staff should never be done in the presence of the client. This undermines the UAP and creates doubt of the staff's competency in the client's mind.

The client diagnosed with a pulmonary embolus is being discharged. Which intervention should the nurse discuss with the client? 1. Increase fluid intake to two (2) to three (3) L/day. 2. Eat a low-cholesterol, low-fat diet. 3. Avoid being around large crowds. 4. Receive pneumonia and flu vaccines.

1 1. Increasing fluids will help increase fluid volume, which will, in turn, help prevent the development of deep vein thrombosis, the most common cause of PE. 2. Pulmonary emboli are not caused by atherosclerosis; this is not an appropriate discharge instruction for a client with a pulmonary embolus. 3. Infection does not cause a PE; this is not an appropriate teaching instruction. 4. Pneumonia and flu do not cause pulmonary embolism.

The client is admitted to a medical unit with a diagnosis of pneumonia. Which signs and symptoms should the nurse assess in the client? 1. Pleuritic chest discomfort and anxiety. 2. Asymmetrical chest expansion and pallor. 3. Leukopenia and CRT <three (3) seconds. 4. Substernal chest pain and diaphoresis.

1 1. Pleuritic chest pain and anxiety from diminished oxygenation occur along with fever, chills, dyspnea, and cough. 2. Asymmetrical chest expansion occurs if the client has a collapsed lung from a pneumothorax or hemothorax, and the client would be cyanotic from decreased oxygenation. 3. The client would have leukocytosis, not leukopenia, and a capillary refill time (CRT) of <3 seconds is normal. 4. Substernal chest pain and diaphoresis are symptoms of myocardial infarction.

Which outcome is appropriate for the client problem "ineffective gas exchange" for the client recently diagnosed with COPD? 1. The client demonstrates the correct way to pursed-lip breathe. 2. The client lists three (3) signs/symptoms to report to the HCP. 3. The client will drink at least 2,500 mL of water daily. 4. The client will be able to ambulate 100 feet with dyspnea.

1 1. Pursed-lip breathing helps keep the alveoli open to allow for better oxygen and carbon dioxide exchange. 2. This would be an appropriate outcome for a knowledge-deficit problem. 3. This outcome does not ensure the client has an effective airway; increasing fluid does not ensure an effective airway. 4. This is not an appropriate outcome for any client problem because the client should be able to ambulate without dyspnea for 100 feet.

The charge nurse is making rounds. Which client should the nurse assess first? 1. The 29-year-old client diagnosed with reactive airway disease who is complaining the nurse caring for him was rude. 2. The 76-year-old client diagnosed with heart failure who has 2+ edema of the lower extremities. 3. The 15-year-old client diagnosed with diabetic ketoacidosis after a bout with the flu who has a blood glucose reading of 189 mg/dL. 4. The 62-year-old client diagnosed with COPD and pneumonia who is receiving O2 by nasal cannula at two (2) liters per minute.

1 1. The charge nurse is responsible for all clients. At times it is necessary to see clients with a psychosocial need before other clients who have expected and non-life-threatening situations. 2. Two (2)+ edema of the lower extremities is expected in a client diagnosed with heart failure. 3. A blood glucose reading of 189 mg/dL is not within normal range, but it is not in a range indicating the client is catabolizing the fats and proteins in the body. No ke- tones will be produced at this blood glucose level, so the ketoacidosis has resolved itself. 4. Most clients diagnosed with COPD are receiving oxygen at a low level.

The nurse is discharging a client newly diagnosed with restrictive airway disease (asthma). Which statement indicates the client understands the discharge instructions? 1. "I will call 911 if my medications don't control an attack." 2. "I should wash my bedding in warm water." 3. "I can still eat at the Chinese restaurant when I want." 4. "If I get a headache, I should take a nonsteroidal anti-inflammatory drug."

1 1. The client must be able to recognize a life-threatening situation and initiate the correct procedure. 2. Bedding is washed in hot water to kill dust mites. 3. Many Chinese dishes are prepared with monosodium glutamate, an ingredient that can initiate an asthma attack. 4. Nonsteroidal anti-inflammatory medica- tions, aspirin, and beta blockers have been known to initiate asthma attacks.

The client diagnosed with an exacerbation of COPD is in respiratory distress. Which intervention should the nurse implement first? 1. Assist the client into a sitting position at 90 degrees. 2. Administer oxygen at six (6) LPM via nasal cannula. 3. Monitor vital signs with the client sitting upright. 4. Notify the health-care provider about the client's status.

1 1. The client should be assisted into a sitting position either on the side of the bed or in the bed. This position decreases the work of breathing. Some clients find it easier sitting on the side of the bed leaning over the bed table. The nurse needs to maintain the client's safety. 2. Oxygen will be applied as soon as possible, but the least amount possible. If levels of oxygen are too high, the client may stop breathing. 3. Vital signs need to be monitored, but this is not the first priority. If the equipment is not in the room, another member of the health-care team should bring it to the nurse. The nurse should stay with the client. 4. The health-care provider needs to be noti- fied, but the client must be treated first. The nurse should get assistance if possible so the nurse can treat this client quickly.

Which information should the nurse teach the client diagnosed with acute sinusitis? 1. Instruct the client to complete all the ordered antibiotics. 2. Teach the client how to irrigate the nasal passages. 3. Have the client demonstrate how to blow the nose. 4. Give the client samples of a narcotic analgesic for the headache.

1 1. The client should be taught to take all antibiotics as ordered. Discontinuing antibiotics prior to the full dose results in the development of antibiotic- resistant bacteria. Sinus infections are difficult to treat and may become chronic, and will then require several weeks of therapy or possibly surgery to control. 2. If the sinuses are irrigated, it is done under anesthesia by a health-care provider. 3. Blowing the nose will increase pressure in the sinus cavities and will cause the client increased pain. 4. The nurse is not licensed to prescribe medications, so this is not in the nurse's scope of practice. Also, narcotic analgesic medications are controlled substances and require written documentation of being prescribed by the health-care provider; samples are not generally available.

The influenza vaccine is in short supply. Which group of clients would the public health nurse consider priority when administering the vaccine? 1. Elderly and chronically ill clients. 2. Child-care workers and children <four (4) years of age. 3. Hospital chaplains and health-care workers. 4. Schoolteachers and students living in a dormitory.

1 1. The elderly and chronically ill are at greatest risk for developing serious complications if they contract the influenza virus. 2. It is recommended people in contact with children receive the flu vaccine whenever possible, but these clients should be able to withstand a bout with the flu if their immune systems are functioning normally. 3. It is probable these clients will be exposed to the virus, but they are not as likely to develop severe complications with intact functioning immune systems. 4. During flu season, the more people the individual comes into contact with, the greater the risk the client will be exposed to the influenza virus, but this group of people would not receive the vaccine before the elderly and chronically ill.

The nurse is assessing a 79-year-old client diagnosed with pneumonia. Which signs and symptoms should the nurse expect to assess in the client? 1. Confusion and lethargy. 2. High fever and chills. 3. Frothy sputum and edema. 4. Bradypnea and jugular vein distention.

1 1. The elderly client diagnosed with pneumonia may present with weakness, fatigue, lethargy, confusion, and poor appetite but not have any of the classic signs and symptoms of pneumonia. 2. Fever and chills are classic symptoms of pneumonia, but they are usually absent in the elderly client. 3. Frothy sputum and edema are signs and symptoms of heart failure, not pneumonia. 4. The client has tachypnea (fast respirations), not bradypnea (slow respirations), and jugular vein distention accompanies heart failure.

The client is suspected of having a pulmonary embolus. Which diagnostic test confirms the diagnosis? 1. Plasma D-dimer test. 2. Arterial blood gases. 3. Chest x-ray. 4. Magnetic resonance imaging.

1 1. The plasma D-dimer test is highly specific for the presence of a thrombus; an elevated D-dimer indicates a thrombus formation and lysis. 2. An ABG evaluates oxygenation level, but it does not diagnose a pulmonary embolus (PE). 3. A CXR shows pulmonary infiltration and pleural effusions, but it does not diagnose a PE. 4. An MRI is a noninvasive test that detects a deep vein thrombosis, but it does

The case manager is arranging a care planning meeting regarding the care of a 65-year-old client diagnosed with adult-onset asthma. Which health-care disciplines should participate in the meeting? Select all that apply. 1. Nursing. 2. Pharmacy. 3. Social work. 4. Occupational therapy. 5. Speech therapy.

1,2,3 1. Nursing is the one discipline remaining with the client around the clock. Therefore, nurses have knowledge of the client that other disciplines might not know. 2. The pharmacist will be able to discuss the medication regimen the client is receiving and make suggestions regarding other medications or medication interactions. 3. The social worker may be able to assist with financial information or home care arrangements. 4. Occupational therapists help clients with activities of daily living and modifications to home environments; nothing in the stem indicates a need for these services. 5. Speech therapists assist clients with speech and swallowing problems; nothing in the stem indicates a need for these services.

Which client problems are appropriate for the nurse to include in the plan of care for the client diagnosed with COPD? Select all that apply. 1. Impaired gas exchange. 2. Inability to tolerate temperature extremes. 3. Activity intolerance. 4. Inability to cope with changes in roles. 5. Alteration in nutrition.

1,2,3,4,5 1. The client diagnosed with COPD has difficulty exchanging oxygen with carbon dioxide, which is manifested by physical signs such as fingernail clubbing and respiratory acidosis as seen on arterial blood gases. 2. The client should avoid extremes in temperatures. Warm temperatures cause an increase in the metabolism and increase the need for oxygen. Cold temperatures cause bronchospasms. 3. The client has increased respiratory effort during activities and can be fatigued. Activities should be timed so rest periods are scheduled to prevent fatigue. 4. The client may have difficulty adapting to the role changes brought about because of the disease process. Many cannot maintain the activities involved in meeting responsibilities at home and at work. Clients should be assessed for these issues. 5. Clients often lose weight because of the effort expended to breathe.

The nurse and a licensed practical nurse (LPN) are caring for five (5) clients on a medical unit. Which clients would the nurse assign to the LPN? Select all that apply. 1. The 32-year-old female diagnosed with exercise-induced asthma who has a forced vital capacity of 1,000 mL. 2. The 45-year-old male with adult-onset asthma who is complaining of difficulty completing all of the ADLs at one time. 3. The 92-year-old client diagnosed with respiratory difficulty who is beginning to be confused and keeps climbing out of bed. 4. The 6-year-old client diagnosed with intrinsic asthma who is scheduled for discharge and the mother needs teaching about the medications. 5. The 20-year-old client diagnosed with asthma who has a pulse oximetry reading of 95% and wants to sleep all the time.

1,2,5 1. A forced vital capacity of 1,000 mL is considered normal for most females; therefore, the LPN could care for this client. 2. The client should be encouraged to pace the activities of daily living; this is expected for a client diagnosed with asthma, so the LPN could care for this client. 3. Confusion could be a sign of decreased oxygen to the brain and requires the RN's expertise. This client should not be assigned to LPN. 4. The client's mother requires teaching, which is the nurse's responsibility and cannot be assigned to a LPN. 5. A pulse oximetry level of 95% is normal, so this client could be assigned to an LPN.

The nurse is caring for the client diagnosed with pneumonia. Which information should the nurse include in the teaching plan? Select all that apply. 1. Place the client on oxygen delivered by nasal cannula. 2. Plan for periods of rest during activities of daily living. 3. Place the client on a fluid restriction of 1,000 mL/day. 4. Restrict the client's smoking to two (2) to three (3) cigarettes per day. 5. Monitor the client's pulse oximetry readings every four (4) hours.

1,2,5 1. The client diagnosed with pneumonia will have some degree of gas-exchange deficit. Administering oxygen would help the client. 2. Activities of daily living require energy and therefore oxygen consumption. Spacing the activities allows the client to rebuild oxygen reserves between activities. 3. Clients are encouraged to drink at least 2,000 mL daily to thin secretions. 4. Cigarette smoking depresses the action of the cilia in the lungs. Any smoking should be prohibited. 5. Pulse oximetry readings provide the nurse with an estimate of oxygenation in the periphery.

Which nursing interventions should the nurse implement for the client diagnosed with a pulmonary embolus who is undergoing thrombolytic therapy? Select all that apply. 1. Keep protamine sulfate readily available. 2. Avoid applying pressure to venipuncture sites. 3. Assess for overt and covert signs of bleeding. 4. Avoid invasive procedures and injections. 5. Administer stool softeners as ordered.

1,3,4,5 1. Heparin is administered during throm- bolytic therapy, and the antidote is prot- amine sulfate and should be available to reverse the effects of the anticoagulant. 2. Firm pressure reduces the risk for bleeding into the tissues. 3. Obvious (overt) as well as hidden (covert) signs of bleeding should be assessed for. 4. Invasive procedures increase the risk of tissue trauma and bleeding. 5. Stool softeners help prevent constipa- tion and straining, which may precipitate bleeding from hemorrhoids.

The nurse is discussing the care of a child diagnosed with asthma with the parent. Which referral is important to include in the teaching? 1. Referral to a dietitian. 2. Referral for allergy testing. 3. Referral to the developmental psychologist. 4. Referral to a home health nurse.

2 1. A child with asthma can eat a regular diet if the child is not allergic to the components of the diet. 2. Because asthma can be a reaction to an allergen, it is important to determine which substances may trigger an attack. 3. The stem did not indicate the child is developmentally delayed. 4. The child does not require a home health nurse solely on the basis of asthma; the school nurse or any child-care provider should be informed of the child's diagnosis, and the parents must know the individual caring for the child is prepared to inter- vene during an attack.

The client diagnosed with asthma is admitted to the emergency department with difficulty breathing and a blue color around the mouth. Which diagnostic test will be ordered to determine the status of the client? 1. Complete blood count. 2. Pulmonary function test. 3. Allergy skin testing. 4. Drug cortisol level.

2 1. A complete blood count determines the oxygen-carrying capacity of the hemoglobin in the body, but it will not identify the immediate problem. 2. Pulmonary function test are completed to determine the forced vital capacity (FVC), the forcedexpiratory capacity in the first second (FEV1), and the peak expiratory flow (PEF). A decline in the FVC, FEV1, and PEF indicates respiratory compromise. 3. Allergy skin testing will be done to determine triggers for allergic asthma, but it is not done during an attack. 4. Drug cortisol levels do not relate to asthma.

The nurse in a long-term care facility is planning the care for a client with a percutaneous endoscopic gastrostomy (PEG) feeding tube. Which intervention should the nurse include in the plan of care? 1. Inspect the insertion line at the naris prior to instilling formula. 2. Elevate the head of the bed after feeding the client. 3. Place the client in the Sims position following each feeding. 4. Change the dressing on the feeding tube every three (3) days.

2 1. A gastrostomy tube is placed directly into the stomach through the abdominal wall; the naris is the opening of the nostril. 2. Elevating the head of the bed uses gravity to keep the formula in the gastric cavity and help prevent it from refluxing into the esophagus, which predisposes the client to aspiration. 3. The Sims position is the left lateral side-lying flat position. This position is used for administering enemas and can be used to prevent aspiration in clients sedated by anesthesia. The sedated client would not have a full stomach. 4. Dressings on PEG tubes should be changed at least daily. If there is no dressing, the insertion site is still assessed daily.

The nurse is assessing the client diagnosed with COPD. Which data require immediate intervention by the nurse? 1. Large amounts of thick white sputum. 2. Oxygen flowmeter set on eight (8) liters. 3. Use of accessory muscles during inspiration. 4. Presence of a barrel chest and dyspnea.

2 1. A large amount of thick sputum is a common symptom of COPD. There is no cause for immediate intervention. 2. The nurse should decrease the oxygen rate to two (2) to three (3) liters. Hypoxemia is the stimulus for breathing in the client with COPD. If the hypoxemia improves and the oxygen level increases, the drive to breathe may be eliminated. 3. It is common for clients with COPD to use accessory muscles when inhaling. These clients tend to lean forward. 4. In clients with COPD, there is a characteristic barrel chest from chronic hyperinflation, and dyspnea is common.

The client diagnosed with influenza A is being discharged from the emergency department with a prescription for antibiotics. Which statement by the client indicates an understanding of this prescription? 1. "These pills will make me feel better fast and I can return to work." 2. "The antibiotics will help prevent me from developing a bacterial pneumonia." 3. "If I had gotten this prescription sooner, I could have prevented this illness." 4. "I need to take these pills until I feel better; then I can stop taking the rest."

2 1. A person with a viral infection should not return to work until the virus has run its course because the antibiotics help prevent complications of the virus, but they do not make the client feel better faster. 2. Secondary bacterial infections often accompany influenza, and antibiotics are often prescribed to help prevent the development of a bacterial infection. 3. Antibiotics will not prevent the flu. Only the flu vaccine will prevent the flu. 4. When people take portions of the antibiotic prescription and stop taking the remainder, an antibiotic-resistant strain of bacteria may develop, and the client may experience a return of symptoms—but this time, the antibiotics will not be effective.

The nurse identified the client problem "decreased cardiac output" for the client diagnosed with a pulmonary embolus. Which intervention should be included in the plan of care? 1. Monitor the client's arterial blood gases. 2. Assess skin color and temperature. 3. Check the client for signs of bleeding. 4. Keep the client in the Trendelenburg position.

2 1. Arterial blood gases would be included in the client problem "impaired gas exchange." 2. These assessment data monitor tissue perfusion, which evaluates for decreased cardiac output. 3. This would be appropriate for the client problem "high risk for bleeding." 4. The client should not be put in a position with the head lower than the legs because this would increase difficulty breathing.

The 56-year-old client diagnosed with tuberculosis (Tb) is being discharged. Which statement made by the client indicates an understanding of the discharge instructions? 1. "I will take my medication for the full three (3) weeks prescribed." 2. "I must stay on the medication for months if I am to get well." 3. "I can be around my friends because I have started taking antibiotics." 4. "I should get a Tb skin test every three (3) months to determine if I am well."

2 1. Clients diagnosed with Tb will need to take the medications for six (6) months to a year. 2. Compliance with treatment plans for Tb includes multidrug therapy for six(6) months to one (1) year for the client to be free of the Tb bacteria. 3. Clients are no longer contagious when three (3) morning sputum specimens are cultured negative, but this will not occur until after several weeks of therapy. 4. The Tb skin test only determines possible exposure to the bacteria, not active disease.

The client is diagnosed with mild intermittent asthma. Which medication should the nurse discuss with the client? 1. Daily inhaled corticosteroids. 2. Use of a "rescue inhaler." 3. Use of systemic steroids. 4. Leukotriene agonists.

2 1. Daily inhaled steroids are used for mild, moderate, or severe persistent asthma, not for intermittent asthma. 2. Clients with intermittent asthma will have exacerbations treated with rescue inhalers. Therefore, the nurse should teach the client about rescue inhalers. 3. Systemic steroids are used frequently by clients with severe persistent asthma, not with mild intermittent asthma. 4. Leukotriene agonists are prescribed for clients diagnosed with mild persistent asthma.

The nurse observes the unlicensed assistive personnel (UAP) removing the nasal cannula from the client diagnosed with COPD while ambulating the client to the bathroom. Which action should the nurse implement? 1. Praise the UAP since this prevents the client from tripping on the oxygen tubing. 2. Place the oxygen back on the client while sitting in the bathroom and say nothing. 3. Explain to the UAP in front of the client oxygen must be left in place at all times. 4. Discuss the UAP's action with the charge nurse so appropriate action can be taken.

2 1. The client diagnosed with COPD needs oxygen at all times, especially when exerting energy such as ambulating to the bathroom. 2. The client needs the oxygen, and the nurse should not correct the UAP in front of the client; it is embarrassing for the UAP and the client loses confidence in the staff. 3. The nurse should not verbally correct a UAP in front of the client; the nurse should correct the behavior and then talk to the UAP in private. 4. The primary nurse should confront the UAP and take care of the situation. Continued unsafe client care would warrant notifying the charge nurse.

The client diagnosed with tonsillitis is scheduled to have surgery in the morning. Which assessment data should the nurse notify the health-care provider about prior to surgery? 1. The client has a hemoglobin of 12.2 g/dL and hematocrit of 36.5%. 2. The client has an oral temperature of 100.2 ̊F and a dry cough. 3. There are one (1) to two (2) white blood cells in the urinalysis. 4. The client's current international normalized ratio (INR) is 1.0.

2 1. The hemoglobin and hematocrit given are within normal range. This would not warrant notifying the health-care provider. 2. A low-grade temperature and a cough could indicate the presence of an infection, in which case the health-care provider would not want to subject the client to anesthesia and the possibility of further complications. The surgery would be postponed. 3. One (1) to two (2) WBCs in a urinalysis is not uncommon because of the normal flora in the bladder. 4. The INR indicates that the client's bleeding time is within normal range.

The nurse is feeding a client diagnosed with aspiration pneumonia who becomes dyspneic, begins to cough, and is turning blue. Which nursing intervention should the nurse implement first? 1. Suction the client's nares. 2. Turn the client to the side. 3. Place the client in Trendelenburg position. 4. Notify the health-care provider.

2 1. The nares are the openings of the nostrils. Suctioning, if done, would be of the posterior pharynx. 2. Turning the client to the side allows for the food to be coughed up and come out of the mouth, rather than be aspirated into the lungs. 3. Placing the client in the Trendelenburg position increases the risk of aspiration. 4. An immediate action is needed to protect the client.

Which nursing assessment data support that the client has experienced a pulmonary embolism? 1. Calf pain with dorsiflexion of the foot. 2. Sudden onset of chest pain and dyspnea. 3. Left-sided chest pain and diaphoresis. 4. Bilateral crackles and low-grade fever.

2 1. This is a sign of a deep vein thrombosis, which is a precursor to a pulmonary em- bolism, but it is not a sign of a pulmonary embolism. 2. The most common signs of a pulmonary embolism are sudden onset of chest pain when taking a deep breath and shortness of breath. 3. These are signs of a myocardial infarction. 4. These could be signs of pneumonia or other pulmonary complications, but not specifically a pulmonary embolism.

Which task is most appropriate for the nurse to delegate to an unlicensed assistive personnel (UAP)? 1. Feed a client who is postoperative tonsillectomy the first meal of clear liquids. 2. Encourage the client diagnosed with a cold to drink a glass of orange juice. 3. Obtain a throat culture on a client diagnosed with bacterial pharyngitis. 4. Escort the client diagnosed with laryngitis outside to smoke a cigarette.

2 1. Tonsillectomies cause throat edema and difficulty swallowing; the nurse must observe the client's ability to swallow before this task can be delegated. 2. Clients with colds are encouraged to drink 2,000 mL of liquids a day. The UAP could do this. 3. Throat swabs for culture must be done correctly or false-negative results can occur. The nurse should obtain the swab. 4. Clients with laryngitis are instructed not to smoke. Smoking is discouraged in all health- care facilities. Sending nursing personnel outside encourages an unhealthy practice, which is not the best use of the personnel.

The employee health nurse is administering tuberculin skin testing to employees who have possibly been exposed to a client with active tuberculosis (Tb). Which statement indicates the need for radiological evaluation instead of skin testing? 1. The client's first skin test indicates a purple flat area at the site of injection. 2. The client's second skin test indicates a red area measuring four (4) mm. 3. The client's previous skin test was read as positive. 4. The client has never shown a reaction to the tuberculin medication.

3 1. A purple flat area indicates that the client became bruised when the intradermal injection was given, but it has no bearing on whether the test is positive. 2. A positive skin test is 10 mm or greater with induration, not redness. 3. If the client has ever reacted positively, then the client should have a chest x-ray to look for causation and inflammation. 4. These are negative findings and do not indicate the need to have x-ray determination of disease.

The nurse is preparing to administer medications to the following clients. Which medication should the nurse question administering? 1. The oral coagulant warfarin (Coumadin) to the client with an INR of 1.9. 2. Regular insulin to a client with a blood glucose level of 218 mg/dL. 3. Hang the heparin bag on a client with a PT/PTT of 12.9/98. 4. A calcium channel blocker to the client with a BP of 112/82.

3 1. An INR of 2 to 3 is therapeutic; therefore, the nurse would administer this medication. 2. This is an elevated blood glucose level; therefore, the nurse should administer the insulin. 3. A normal PTT is 39 seconds, and for heparin to be therapeutic, it should be 1.5 to 2 times the normal value, or 58 to 78. A PTT of 98 indicates the client is not clotting and the medication should be held. 4. This is a normal blood pressure and the nurse should administer the medication.

The client diagnosed with a community-acquired pneumonia is being admitted to the medical unit. Which nursing intervention has the highest priority? 1. Administer the ordered oral antibiotic STAT. 2. Order the meal tray to be delivered as soon as possible. 3. Obtain a sputum specimen for culture and sensitivity. 4. Have the unlicensed assistive personnel weigh the client.

3 1. Broad-spectrum IV antibiotics are priority, but before antibiotics are administered, it is important to obtain culture specimens to determine the correct antibiotic for the client's infection. Clients are placed on oral medications only after several days of IVPB therapy. 2. Meal trays are not priority over cultures. 3. To determine the antibiotic that will effectively treat an infection, specimens for culture are taken prior to beginning the medication. Administering antibi- otics prior to cultures may make it impossible to determine the actual agent causing the pneumonia. 4. Admission weights are important to determine appropriate dosing of medication, but they are not priority over sputum collection.

Which clinical manifestation should the nurse expect to assess in the client recently diagnosed with COPD? 1. Clubbing of the client's fingers. 2. Infrequent respiratory infections. 3. Chronic sputum production. 4. Nonproductive hacking cough.

3 1. Clubbing of the fingers is the result of chronic hypoxemia, which is expected with chronic COPD but not recently diagnosed COPD. 2. These clients have frequent respiratory infections. 3. Sputum production, along with cough and dyspnea on exertion, are the early signs/symptoms of COPD. 4. These clients have a productive cough, not a nonproductive cough.

The nurse is completing the admission assessment on a 13-year-old client diagnosed with an acute exacerbation of asthma. Which signs and symptoms would the nurse expect to find? 1. Fever and crepitus. 2. Rales and hives. 3. Dyspnea and wheezing. 4. Normal chest shape and eupnea.

3 1. Fever is a sign of infection, and crepitus is air trapped in the layers of the skin. 2. Rales indicate fluid in the lung, and hives are a skin reaction to a stimulus such as occurs with an allergy to a specific substance. 3. During an asthma attack, the muscles surrounding the bronchioles constrict, causing a narrowing of the bronchioles. The lungs then respond with the production of secretions that further narrow the lumen. The resulting symptoms include wheezing from air passing through the narrow, clogged spaces, and dyspnea. 4. During an attack, the chest will be expanded from air being trapped and not being exhaled. A chest x-ray will reveal a lowered diaphragm and hyperinflated lungs.

The home health-care nurse is talking on the telephone to a male client diagnosed with hypertension and hears the client sneezing. The client tells the nurse he has been blowing his nose frequently. Which question should the nurse ask the client? 1. "Have you had the flu shot in the last two (2) weeks?" 2. "Are there any small children in the home?" 3. "Are you taking over-the-counter medicine for these symptoms?" 4. "Do you have any cold sores associated with your sneezing?"

3 1. Influenza is a viral illness that might cause these symptoms; however, an immunization should not give the client the illness. 2. Coming into contact with small children increases the risk of contracting colds and the flu, but the client has a problem—not just a potential one. 3. A client diagnosed with hypertension should not take many of the over-the- counter medications because they work by causing vasoconstriction, which will increase the hypertension. 4. Cold sores are actually an infection by the herpes simplex virus. Colds and cold-like symptoms are caused by the rhinovirus or influenza virus. The term "cold sore" is a common term that still persists in the populace.

The client has been diagnosed with chronic sinusitis. Which sign/symptom alerts the nurse to a potentially life-threatening complication? 1. Muscle weakness. 2. Purulent sputum. 3. Nuchal rigidity. 4. Intermittent loss of muscle control.

3 1. Muscle weakness is a sign/symptom of myalgia, but it is not a life-threatening complication of sinusitis. 2. Purulent sputum would be a sign/symptom of a lung infection, but it is not a life-threatening complication of sinusitis. 3. Nuchal rigidity is a sign/symptom of meningitis, which is a life-threatening potential complication of sinusitis resulting from the close proximity of the sinus cavities to the meninges. 4. Intermittent loss of muscle control can be a symptom of multiple sclerosis, but it would not be a life-threatening complication of sinusitis.

The client diagnosed with exercise-induced asthma (EIA) is being discharged. Which information should the nurse include in the discharge teaching? 1. Take two (2) puffs on the rescue inhaler and wait five (5) minutes before exercise. 2. Warm-up exercises will increase the potential for developing the asthma attacks. 3. Use the bronchodilator inhaler immediately prior to beginning to exercise. 4. Increase dietary intake of food high in monosodium glutamate (MSG).

3 1. Rescue inhalers are used to treat attacks, not prevent them, so this should not be administered prior to exercising. 2. Warm-up exercises decrease the risk of developing an asthma attack. 3. Using a bronchodilator immediately prior to exercising will help reduce bronchospasms. 4. Monosodium glutamate, a food preservative, has been shown to initiate asthma attacks.

The client diagnosed with a pulmonary embolus is in the intensive care unit. Which assessment data warrant immediate intervention from the nurse? 1. The client's ABGs are pH 7.36, PaO2 95, PaCO2 38, HCO3 24. 2. The client's telemetry exhibits occasional premature ventricular contractions. 3. The client's pulse oximeter reading is 90%. 4. The client's urinary output for the 12-hour shift is 800 mL.

3 1. The ABGs are within normal limits and would not warrant immediate intervention. 2. Occasional premature ventricular contrac- tions are not unusual for any client and would not warrant immediate intervention. 3. The normal pulse oximeter reading is 93% to 100%. A reading of 90% indicates the client has an arterial oxygen level of around 60. 4. A urinary output of 800 mL over 12 hours indicates an output of greater than 30 mL/hour and would not warrant immediate intervention by the nurse.

Which referral is most appropriate for a client diagnosed with end-stage COPD? 1. The Asthma Foundation of America. 2. The American Cancer Society. 3. The American Lung Association. 4. The American Heart Association.

3 1. The Asthma Foundation of America is not appropriate for a client in this stage of COPD. 2. The American Cancer Society is helpful for a client with lung cancer but not for a client with COPD. 3. The American Lung Association has information helpful for a client with COPD. 4. Many clients with COPD end up with heart problems, but the American Heart Association does not have information for clients with COPD.

The nurse is developing a plan of care for a client diagnosed with laryngitis and identifies the client problem "altered communication." Which intervention should the nurse implement? 1. Instruct the client to drink a mixture of brandy and honey several times a day. 2. Encourage the client to whisper instead of trying to speak at a normal level. 3. Provide the client with a blank note pad for writing any communication. 4. Explain that the client's aphonia may become a permanent condition.

3 1. The client with laryngitis is instructed to avoid all alcohol. Alcohol causes increased irritation of the throat. 2. Whispering places added strain on the larynx. 3. Voice rest is encouraged for the client experiencing laryngitis. 4. Aphonia, or inability to speak, is a tempo- rary condition associated with laryngitis.

The nurse is preparing to administer the oral anticoagulant warfarin (Coumadin) to a client who has a PT/PTT of 22/39 and an INR of 2.8. What action should the nurse implement? 1. Assess the client for abnormal bleeding. 2. Prepare to administer vitamin K (AquaMephyton). 3. Administer the medication as ordered. 4. Notify the HCP to obtain an order to increase the dose.

3 1. The client would not be experiencing abnormal bleeding with this INR. 2. This is the antidote for an overdose of anticoagulant and the INR does not indicate this. 3. A therapeutic INR is 2 to 3; therefore, the nurse should administer the medication. 4. There is no need to increase the dose; this result is within the therapeutic range.

The nurse is caring for the client diagnosed with COPD. Which outcome requires a revision in the plan of care? 1. The client has no signs of respiratory distress. 2. The client shows an improved respiratory pattern. 3. The client demonstrates intolerance to activity. 4. The client participates in establishing goals.

3 1. The expected outcome showing no signs of respiratory distress indicates the plan of care is effective and should be continued. 2. An improved respiratory pattern indicates the plan should be continued. 3. The expected outcome should be that the client has tolerance for activity; because the client is not meeting the expected outcome, the plan of care needs revision. 4. The client should participate in planning the course of care. The client is meeting the expected outcome.

The day shift charge nurse on a medical unit is making rounds after report. Which client should be seen first? 1. The 65-year-old client diagnosed with tuberculosis who has a sputum specimen to be sent to the lab. 2. The 76-year-old client diagnosed with aspiration pneumonia who has a clogged feeding tube. 3. The 45-year-old client diagnosed with pneumonia who has a pulse oximetry reading of 92%. 4. The 39-year-old client diagnosed with bronchitis who has an arterial oxygenation level of 89%.

3 1. The specimen needs to be taken to the laboratory within a reasonable time frame, but a UAP can take specimens to the laboratory. 2. Clogged feeding tubes occur with some regularity. Delay in feeding a client will not result in permanent damage. 3. A pulse oximetry reading of 92% means that the arterial blood oxygen saturation is somewhere around 60% to 70%. 4. Arterial oxygenation normal values are 80% to 100%.

The charge nurse on a surgical floor is making assignments. Which client should be assigned to the most experienced registered nurse (RN)? 1. The 36-year-old client who has undergone an antral irrigation for sinusitis yesterday and has moderate pain. 2. The six (6)-year-old client scheduled for a tonsillectomy and adenoidectomy this morning who will not swallow medication. 3. The 18-year-old client who had a Caldwell-Luc procedure three (3) days ago and has purulent drainage on the drip pad. 4. The 45-year-old client diagnosed with a peritonsillar abscess who requires IVPB antibiotic therapy four (4) times a day.

3 1. This client is one (1) day postoperative and has moderate pain, which is to be expected after surgery. A less experienced nurse can care for this client. 2. A child about to go to surgery involving the throat area can be expected to have painful swallowing. This does not require the most experienced nurse. 3. The postoperative client with purulent drainage could be developing an infection. The experienced nurse would be needed to assess and monitor the client's condition. 4. Any nurse who is capable of administering IVPB medications can care for this client.

Which assessment data best indicate the client with Type 1 diabetes is adhering to the medical treatment regimen? 1. The client's fasting blood glucose is 100 mg/dL. 2. The client's urine specimen has no ketones. 3. The client's glycosylated hemoglobin is 5.8%. 4. The client's glucometer reading is 120 mg/dL.

3. A glycosylated hemoglobin (A1C) gives the average of the blood glucose level over the last 3 months and indicates adherence to the medical treatment regi- men. A glycosylated hemoglobin level of 5.8% is close to normal and indicates that the client is adhering to the treatment regimen. The following table shows blood glucose levels and corresponding glycosylated hemoglobin results: Blood GlucoseLevel 70-110 135 170 205 240 275 310 345Glycosylated Hemo- globin Result 4.0-5.5% Normal 6% 7% 8% 9% 10% 11% 12%

Which statement indicates to the nurse the client diagnosed with asthma understands the teaching regarding mast cell stabilizer medications? 1. "I should take two (2) puffs when I begin to have an asthma attack." 2. "I must taper off the medications and not stop taking them abruptly." 3. "These drugs will be most effective if taken at bedtime." 4. "These drugs are not good at the time of an attack."

4 1. Mast cell stabilizers require 10 to 14 days to be effective. Some clients diagnosed with exercise-induced asthma derive benefit from taking the drugs immediately before exercising, but these drugs must be in the system for a period of time before effectiveness can be achieved. 2. Tapering of medications is done for systemic steroids because of adrenal functioning. 3. The drugs are taken daily, before exercise, or both. 4. Mast cell drugs are routine maintenance medications and do not treat an attack.

The nurse is planning the care of a client diagnosed with pneumonia and writes a problem of "impaired gas exchange." Which is an expected outcome for this problem? 1. Performs chest physiotherapy three (3) times a day. 2. Able to complete activities of daily living. 3. Ambulates in the hall several times during each shift. 4. Alert and oriented to person, place, time, and events.

4 1. Clients do not perform chest physiotherapy; this is normally done by the respiratory therapist. This is a staff goal, not a client goal. 2. This would be a goal for self-care deficit but not for impaired gas exchange. 3. This would be a goal for the problem of activity intolerance. 4. Impaired gas exchange results in hypoxia, the earliest sign/symptom of which is a change in the level of consciousness.

The school nurse is presenting a class to students at a primary school on how to prevent the transmission of the common cold virus. Which information should the nurse discuss? 1. Instruct the children to always keep a tissue or handkerchief with them. 2. Explain that children current with immunizations will not get a cold. 3. Tell the children they should go to the doctor if they get a cold. 4. Demonstrate to the students how to wash hands correctly.

4 1. It is not feasible for a child to always have a tissue or handkerchief available. 2. There is no immunization for the common cold. Colds are actually caused by at least 200 separate viruses and the viruses mutate frequently. 3. Colds are caused by a virus and antibiotics do not treat a virus; therefore, there is no need to go to a health-care provider. 4. Hand washing is the single most useful technique for prevention of disease.

The client is admitted with a diagnosis of rule-out tuberculosis. Which type of isolation procedures should the nurse implement? 1. Standard Precautions. 2. Contact Precautions. 3. Droplet Precautions. 4. Airborne Precautions.

4 1. Standard Precautions are used to prevent exposure to blood and body secretions on all clients. Tuberculosis is caused by airborne bacteria. 2. Contact Precautions are used for wounds. 3. Droplet Precautions are used for infections spread by sneezing or coughing but not transmitted over distances of more than three (3) to four (4) feet. 4. Tuberculosis bacteria are capable of disseminating over long distances on air currents. Clients with tuberculosis are placed in negative air pressure rooms where the air in the room is not allowed to cross-contaminate the air in the hallway.

The client diagnosed with chronic sinusitis who has undergone a Caldwell-Luc procedure is complaining of pain. Which intervention should the nurse implement first? 1. Administer the narcotic analgesic IVP. 2. Perform gentle oral hygiene. 3. Place the client in semi-Fowler's position. 4. Assess the client's pain.

4 1. The client has complained of pain, and the nurse, after determining the severity of the pain and barring any complications in the client, will administer pain medica- tion after completion of the assessment. 2. Oral hygiene helps to prevent the development of infections and promotes comfort, but it will not relieve the pain. 3. Placing the client in the semi-Fowler's position will reduce edema of inflamed sinus tissue, but it will not immediately affect the client's perception of pain. 4. Prior to intervening, the nurse must assess to determine the amount of pain and possible complications occurring that could be masked if narcotic medication is administered.

Which statement by the client diagnosed with a pulmonary embolus indicates the discharge teaching is effective? 1. "I am going to use a regular-bristle toothbrush." 2. "I will take antibiotics prior to having my teeth cleaned." 3. "I can take enteric-coated aspirin for my headache." 4. "I will wear a Medic Alert band at all times."

4 1. The client should use a soft-bristle toothbrush to reduce the risk of bleeding, so the teaching is not effective. 2. This is appropriate for a client with a mechanical valve replacement, not a client receiving anticoagulant therapy, so the teaching is not effective. 3. Aspirin, enteric-coated or not, is an antiplatelet, which may increase bleeding tendencies and should be avoided, so the teaching is not effective. 4. The client should wear a Medic Alert band at all times so that, if any accident or situation occurs, the health-care providers will know the client is receiving anticoagulant therapy. The client understands the teaching.

The nurse is caring for the client diagnosed with end-stage COPD. Which data warrant immediate intervention by the nurse? 1. The client's pulse oximeter reading is 92%. 2. The client's arterial blood gas level is 74. 3. The client has SOB when walking to the bathroom. 4. The client's sputum is rusty colored.

4 1. The client with end-stage COPD has decreased peripheral oxygen levels; therefore, this would not warrant immediate intervention. 2. The client's ABGs would normally indicate a low oxygen level; therefore, this would not warrant immediate intervention. 3. The client who develops dyspnea on exertion should stop the exertion but does not require intervention by the nurse if the dyspnea resolves. 4. Rusty-colored sputum indicates blood in the sputum and requires further assessment by the nurse.

The client is admitted to the medical unit diagnosed with a pulmonary embolus. Which intervention should the nurse implement? 1. Administer oral anticoagulants. 2. Assess the client's bowel sounds. 3. Prepare the client for a thoracentesis. 4. Institute and maintain bedrest.

4 1. The intravenous anticoagulant heparin will be administered immediately after diagnosis of a PE, not oral anticoagulants. 2. The client's respiratory system will be assessed, not the gastrointestinal system. 3. A thoracentesis is used to aspirate fluid from the pleural space; it is not a treatment for a PE. 4. Bedrest reduces the risk of another clot becoming an embolus leading to a pulmonary embolus. Bedrest reduces metabolic demands and tissue needs for oxygen in the lungs.

The nurse is assessing the client with COPD. Which health promotion information is most important for the nurse to obtain? 1. Number of years the client has smoked. 2. Risk factors for complications. 3. Ability to administer inhaled medication. 4. Willingness to modify lifestyle.

4 1. The number of years of smoking is infor- mation needed to treat the client but not the most important in health promotion. 2. The risk factors for complications are important in planning care. 3. Assessing the ability to deliver medications is an important consideration when teaching the client. 4. The client's attitude toward lifestyle changes is the most important consideration in health promotion, in this case smoking cessation. The nurse should assess if the client is willing to consider cessation of smoking and carry out the plan.

Which statement made by the client diagnosed with chronic bronchitis indicates to the nurse more teaching is required? 1. "I should contact my health-care provider if my sputum changes color or amount." 2. "I will take my bronchodilator regularly to prevent having bronchospasms." 3. "This metered-dose inhaler gives a precise amount of medication with each dose." 4. "I need to return to the HCP to have my blood drawn with my annual physical."

4 1. When sputum changes in color, amount, or both, this indicates infection, and the client should report this information to the health-care provider. This statement indi- cates the client understands the teaching. 2. Bronchodilators should be taken routinely to prevent bronchospasms. This statement indicates the client understands the teaching. 3. Clients use metered-dose inhalers because they deliver a precise amount of medication with correct use. This statement indicates the client understands the teaching. 4. Clients should have blood levels drawn every six (6) months when taking bron- chodilators, not yearly. This indicates the client needs more teaching.

A client with significant diarrhea has arterial blood gases drawn. The results are pH 7.50, PCO2 35, HCO3 30. Which nursing interpretation of the client's acid-base imbalance is correct? 1. Respiratory alkalosis 2. Metabolic alkalosis 3. Respiratory acidosis 4. Metabolic acidosis

ANSWER: 2 A pH of 7.50 indicates alkalosis. A corresponding HCO3 above the normal range of 20 to 24 mmol/L indicates the primary acid-base im- balance is metabolic alkalosis. A respiratory alkalosis would result in a PCO2 lower than the normal range of 35 to 45 mm Hg. Respiratory acidosis and metabolic acidosis would result in a pH less than 7.4.

A 17-year-old client with cystic fibrosis (CF) is visiting with a nurse in preparation for leaving home for college. The nurse knows that the client needs fur- ther education if the client states: 1. "I will bring extra cough medicine so as to not wake up my roommate at night." 2. "I will contact the college's health center and pass on my medical records." 3. "I will check to make sure they have good work- out facilities." 4. "I will be really careful about washing my hands and staying away from sick friends."

ANSWER: 1 A client with CF should not be using cough suppressants because of the high importance of expectorating secretions. Letting the health cen- ter know of special care needs and familiarizing them with the client can help begin the therapeutic relationship and reduce the anxieties of both the client and the care providers. It is important for clients with CF to exercise regularly and maintain physical strength in order to facilitate loosening and clearing secretions. Hand hygiene and avoiding sick friends will help to re- duce the risk of respiratory illness.

A nurse is caring for a client suspected of having a pulmonary embolism. The client's arterial blood gas (ABG) results indicate respiratory alkalosis. Which findings support this diagnosis? 1.pH = 7.54; PaCO2 = 25; HCO3 = 24 2. pH = 7.35; PaCO2 = 35; HCO3 = 22 3.pH = 7.50; PaCO2 = 40; HCO3 = 28 4.pH = 7.32; PaCO2 = 48; HCO3 = 24

ANSWER: 1 Because pulmonary emboli interfere with gas exchange, the respiratory center is stimulated to meet oxygenation demands. The tachypnea pro- duces respiratory alkalosis. Thus, the pH is increased above normal of 35 to 45 and the PaCO2 is lower than the normal level of 35 to 45 mm Hg. The HCO3 is normally 22 to 26 mEq/L. The blood gas in option 2 is normal, option 3 represents metabolic alkalosis, and option 4 is indicative of respiratory acidosis.

A client with type 1 diabetes mellitus is scheduled for a total hip replacement. In reviewing the client's or- ders the evening prior to surgery, a nurse notes that the physician did not write an order to change the client's daily insulin dose. Which nursing action is most appropriate? 1. Notify the physician who wrote the insulin order in the client's medical record. 2. Write an order to decrease the morning insulin dose by one-half of the prescribed morning dose. 3. Do nothing because the physician would want the client to receive the usual insulin dose prior to surgery. 4. Inform the day shift nurse to check the client's fingerstick glucose before surgery and hold the morning dose of insulin.

ANSWER: 1 Because the client will be NPO (nothing per mouth) for surgery, the nurse must ensure that the usual insulin dosage has been changed to prevent hypoglycemia. The change may include eliminating the rapid- acting insulin and giving a decreased amount of intermediate-acting NPH or Lente insulin. A registered nurse is unable to prescribe medica- tions. The nurse could write the order based on standing orders, but this is not noted in the option. Doing nothing could cause a hypoglycemic reac- tion because the client will be NPO for surgery. Holding the morning dose of insulin can cause hyperglycemia leading to diabetic ketoacidosis. Even without food, glucose levels increase from hepatic glucose production. Clients with type 1 diabetes require insulin 24 hours a day.

Which physician's order should the nurse question for a newly admitted client diagnosed with diabetic ketoacidosis (DKA)? 1. D5W at 125 mL per hour 2.KCL 10 mEq in 100 mL NaCl IV now 3. Stat arterial blood gases. Administer sodium bicarbonate if pH is less than 7.0. 4. Regular insulin infusion per protocol adjusting dose based on hourly glucose levels

ANSWER: 1 In DKA, the blood glucose level is above 300 mg/dL. Additional glu- cose will only increase the glucose level. Initially 0.45% or 0.9% sodium chloride (NaCl) is administered for fluid resuscitation. Glucose may be added when blood glucose levels approach 250 mg/dL. Insulin will drive potassium into the cells, so potassium chloride (KCL) is admin- istered to prevent life-threatening hypokalemia. Normal pH is 7.35 to 7.45. Sodium bicarbonate will reverse the severe acidosis. Intravenous (IV) in- sulin will correct the hyperglycemia and hyperketonemia. Tight glucose control can be maintained by hourly glucose checks and adjusting the insulin infusion dose.

A nurse is working with a client to update the client's asthma action plan. The nurse knows that this action plan should include information on: 1. medication adjustments that should be made if peak flow is less than 50% normal. 2. timeline for allergy skin testing. 3. the most direct route when the client drives to the hospital. 4. the best methods for chest physiotherapy (CPT).

ANSWER: 1 The asthma action plan is intended to help clients determine how and when to adjust care if their asthma worsens; primarily through adjustment of medication regimen. The plan also identifies the best ways to access and alert emergency personnel if an acute attack occurs but the client should not be driving him- or herself to the hospital. Allergy skin testing would be done in the early phases of diagnosis. CPT is not usually a part of asthma therapy.

A home health nurse is visiting a client whose chronic bronchitis has recently worsened. Which in- struction should the nurse reinforce with this client? 1. Increase amount of bedrest 2. Increase fluid intake 3. Decrease caloric intake 4. Reduce home oxygen use

ANSWER: 2 Adequate fluids may help liquefy secretions for easier expectoration. Imposing bedrest on a client with shortness of breath may worsen the situ- ation. Physical activity interspersed with adequate rest can improve respi- ratory functioning. A diet high in calories can compensate for these client's hypermetabolic state, dyspnea, and poor appetite. Reducing home oxygen use, in this situation, would most likely exacerbate the client's symptoms.

A nurse is teaching an elderly client about the impor- tance of using the item that is attached to the inhaler. The nurse should explain that this item: 1. allows for a greater amount of medication to be delivered. 2. lets the client see the medication as it is delivered. 3. keeps the mouthpiece sterile. 4. allows for activating the medication canister by simply inhaling.

ANSWER: 1 The item pictured is a spacer or extender. These devices hold the medication in suspension for a few moments longer, allowing improved medication delivery. These devices are recommended for children or older adults who might not have the dexterity or knowledge to time an inhalation with medication delivery. Visual confirmation of delivery is not the main reason that the device is attached. The mouthpiece may be kept cleaner with a spacer but does not ensure sterility. Although some in- halers allow for inhalation "trigger" delivery, this is not the purpose of the spacer/extender. ➧ Test-taking Tip: If unfamiliar with the item pictured, think logically about the situation and link it to the aspects of the question that you do know. In this case, some of the options do not seem to fit with the functions of what appears in the picture.

A home-health nurse is planning the first home visit for a 60-year-old Hispanic client newly diag- nosed with type 2 diabetes mellitus. The client has been instructed to take 70/30 combination insulin in the morning and at suppertime. Which interventions should be included in the client's plan of care? SELECT ALL THAT APPLY. 1. Instruct the client to inspect the feet daily. 2. Ensure that the client eats a bedtime snack. 3. Assess the client's ability to read small print. 4. Teach the client how to perform a hemoglobin A1c test. 5. Instruct the client on storing prefilled syringes in the refrigerator. 6. Teach the client to take one unit of 70/30 insulin after eating a snack.

ANSWER: 1, 2, 3, 5 Diabetes, diabetic complications, and increased mortality have been reported to occur at a higher rate in Hispanics compared with non- Hispanic whites of the same age. Therefore, careful daily skin assess- ment is necessary. Neuropathy, peripheral vascular disease, and im- munocompromise can result in diabetic foot ulcer and complications. The 70/30 insulin is a combination of NPH and regular insulin. The NPH insulin will peak 4 to 12 hours after administration. A bedtime snack will cover the insulin peak to prevent hypoglycemia. Magnifying devices are available if the client is unable to read small print to pre- vent dosing errors. Syringes may be prefilled but should be stored in the refrigerator with the needle up. Blood is drawn in the laboratory to check the A1c. Only short-acting (regular) or rapid-acting insulin (aspart or lispro), not 70/30 insulin, would be administered to cover for additional carbohydrates if the client were on a carbohydrate-counting regimen with insulin coverage. This would be prescribed by the physician.

Which instructions should the nurse provide to a client regarding diabetes management during stress or illness? SELECT ALL THAT APPLY. 1. Notify the health-care provider if unable to keep fluids or foods down. 2. Test fingerstick glucose levels and urine ketones daily and keep a record. 3. Continue to take oral hypoglycemic medications and/or insulin as prescribed. 4. Supplement food intake with carbohydrate- containing fluids, such as juices or soups. 5. When on an oral agent, administer insulin in addition to the oral agent during the illness. 6. A minor illness, such as the flu, usually does not affect the blood glucose and insulin needs.

ANSWER: 1, 3 An acute or minor illness can evoke a counterregulatory hormone re- sponse resulting in hyperglycemia, thus the client should continue medications as prescribed. If the client is unable to eat due to nausea and vomiting, dehydration can occur from hyperglycemia and the lack of fluid intake. Blood glucose should be checked every 4 hours when ill and the ketones tested every 3 to 4 hours if the glucose is greater than 240 mg/dL. The client should supplement the diet with carbohydrate-con- taining fluids only if eating less than normal due to the illness. Insulin may or may not be necessary; it is based on the client's blood glucose level.

A public health nurse is planning a flu shot clinic. The nurse is working on advertising. Which groups should be the highest priority to target when advertising the flu shot clinic? SELECT ALL THAT APPLY. 1. Pregnant women 2. Grade school children 3. Nursing assistants at a nursing home 4. A hypertension clinic population 5. Outpatient psychiatric population 6. Spinal cord-injured population at an assisted living facility

ANSWER: 1, 3, 6 The Centers for Disease Control and Prevention (CDC) provides guide- lines for identifying those clients at high risk for influenza-related com- plications and severe disease. These groups include children aged 6 to 59 months, pregnant women, and clients older than 50 years of age. Those who have chronic respiratory conditions, their caregivers and household contacts, and those in the health-care field are also consid- ered high priorities for vaccination. Clients with spinal cord injury are at high risk for respiratory complications and are prone to complica- tions of flu related to immobilization and difficulty with clearing secre- tions. Grade school children, clients with hypertension, and persons receiv- ing outpatient psychiatric care are not considered high risk by the CDC.

A client requires intravenous vancomycin (Vancocin®) for antibiotic-resistant pneumonia. The order calls for 500 mg to be administered, and the medication is supplied in a 100 mL piggyback that contains 5 mg per 1 mL to run over 1 hour. In order to administer the correct dose, a nurse should set the infusion pump to run at a rate of _____ mL per hour.

ANSWER: 100 Use the formula for calculating intravenous flow rates. Formula: Infuse rate (in mL/hour) = Volume to be infused Infusion time 100 mL/hr = 100 mL 1 hr

A client, newly diagnosed with asthma is preparing for discharge. Which point should a nurse emphasize during the client's teaching? 1. Contact care provider only if nighttime wheezing becomes a concern 2. Limit exposure to sources that trigger an attack 3. Use peak flow meter only if symptoms are worsening 4. Use inhaled steroid medication as a rescue inhaler

ANSWER: 2 A client newly diagnosed with asthma has a large number of educa- tional concerns that need to be addressed. Of primary importance is knowing ways to prevent an attack, such as avoiding known triggers. A peak flow meter is generally used on a daily basis to help document and identify worsening symptoms over time. Symptoms, such as worsening peak flow meter readings and nighttime wheezing, are one of many health changes that should signal the client to contact his or her care provider. Generally speaking, inhalers with steroid medications are not to be used as a rescue inhaler in the event of an attack.

A nurse is teaching a client who has been newly diagnosed with type 2 diabetes mellitus (DM). Which teaching point should the nurse emphasize? 1. Use the arm when self-administering NPH insulin. 2. Exercise for 30 minutes daily, preferably after a meal. 3. Consume 30% of the daily calorie intake from protein foods. 4. Eat a 30-gram carbohydrate snack prior to strenuous activity.

ANSWER: 2 Exercise increases insulin receptor sites in the tissue and can have a direct effect on lowering blood glucose levels. Exercise also contributes to weight loss, which also decreases insulin resistance. Usually type 2 DM is controlled with oral hypoglycemic agents. If insulin is needed, sites should be rotated. For those with DM, protein should contribute less than 10% of the total energy consumed. Strenuous activity can be perceived by the body as a stressor, causing a release of counterregulatory hormones that subsequently increases blood glucose. Hyperglycemia can result from the combination of strenuous activity and extra carbohydrates.

A nurse begins to hear high-pressure alarms in the room of a client requiring respiratory assistance with a ventilator. Which is the best action by the nurse? 1. Wait and allow the client time to regulate breathing in coordination with the ventilator 2. Check ventilator tubing and connections 3. Silence the alarm and restart the ventilator 4. Lower the tidal volumes being delivered to the client

ANSWER: 2 High-pressure alarms should be immediately investigated by the nurse. Tubing and connections are often a source of both high- and low-pressure alarms. The causes may include the client biting on the endotracheal tube or fighting the ventilator-delivered breaths or mi- gration of the endotracheal tube, which can cause coughing. Changing the ventilator settings would not change the cause of the problem if it is in- deed external to the client. Waiting or silencing alarms is a threat to client safety.

A nurse evaluates a client who is being treated for diabetic ketoacidosis (DKA). Which finding indicates that the client is responding to the treatment plan? 1. Eyes sunken, skin flushed 2. Skin moist with rapid elastic recoil 3. Serum potassium level is 3.3 mEq/L 4. ABG results are pH 7.25, PaCO2 30, HCO3 17

ANSWER: 2 Moist skin and good skin turgor indicate that dehydration secondary to hyperglycemia is resolving. Sunken eyes and flushing are signs of dehydration. Normal serum potassium levels are 3.5 to 5.8 mEq/L. The abnormal ABGs indicate compensating metabolic acidosis. ➧ Test-taking Tip: Note the key phrase "responding to treatment." Select the option that is a normal finding.

A nurse is working at a telephone health service. Which advice should the nurse give to a client who has had 3 days of symptoms that strongly suggest influenza? 1. Return to work after another day of rest 2. Rest and increase fluid intake to 3 liters of fluid per day 3. Use over-the-counter antihistamines 4. Make an appointment to get the flu shot

ANSWER: 2 Rest and increased fluid intake are essential. Influenza is generally a self-limiting condition, but one that is also highly contagious, so returning to work should not occur until the client is symptom-free. Antihistamines are not the over-the-counter medications of choice. Generally antitussives and antipyretics are optimal for symptom control. Clients who have an ac- tive case of influenza should not get the shot.

An 80-year-old client is living in an inde- pendent living facility with home health nursing support. The client is diagnosed with pneumonia and started on an oral antibiotic. Which nursing di- agnosis would be most appropriate for this client? 1. Risk for imbalanced nutrition 2. Risk for fluid volume deficit 3. Fluid volume deficit 4. Fluid volume excess

ANSWER: 2 The diagnosis of pneumonia may result in fever or increased respira- tory rate that increases amount of fluid lost. Additionally, older adults have a decreased sensation of thirst. Nutrition may be affected due to a diagnosis of pneumonia, but fluid volume would be the greatest concern with pneumonia. The client's age and a diagnosis of pneumonia could result in a fluid volume deficit, but there is no information to support that the client is deficient in fluid. There is no information to support an excess fluid volume.

A nurse is partnered with a patient care assistant (PCA) on a medical-surgical floor. The PCA provides information about the clients for whom the PCA has been caring. Based on the information from the PCA, which client should the nurse attend to first? 1. The client with a pulmonary embolus who has not had a bowel movement in 2 days 2. The client who underwent a video thoracoscopy with oxygen saturation readings from 88% to 90% on oxygen at 4 L/NC 3. The client who underwent a wedge resection of right lung and has a blood pressure of 100/65 mm Hg 4. The client who has rib fractures and has not voided for 6 hours after the urinary catheter was removed

ANSWER: 2 The most concerning report from the PCA is regarding the client who is not maintaining oxygen saturations despite receiving oxygen. None of the other clients have potentially life-threatening conditions or concerns that could not later be addressed. Although the blood pressure is low in option 3, it is only one data point and obtaining a repeat reading should be delegated to the PCA.

A nurse administers 15 units of glargine (Lantus®) insulin at 2100 hours to a Hispanic client when the client's fingerstick blood glucose reading was 110 mg/dL. At 2300 hours, a nursing assistant reports to the nurse that an evening snack was not given because the client was sleeping. Which instruc- tion by the nurse is most appropriate? 1. "You will need to wake the client to check the blood glucose and then give a snack. All diabetics get a snack at bedtime." 2. "It is not necessary for this client to have a snack because glargine insulin is absorbed very slowly over 24 hours and doesn't have a peak." 3. "The next time the client wakes up, check a blood glucose level and then give a snack." 4. "I will need to notify the physician because a snack at this time will affect the client's blood glucose level and the next dose of glargine insulin."

ANSWER: 2 The onset of glargine is 1 hour, it has no peak, and it lasts for 24 hours. Glargine lowers the blood glucose by increasing transport into cells and promoting the conversion of glucose to glycogen. Because it is peakless, a bedtime snack is unnecessary. Options 1 and 3 are unnecessary and option 4 is incorrect. Glargine is administered once daily, the same time each day, to maintain relatively constant concentrations over 24 hours.

A nurse administers a usual morning dose of 4 units of regular insulin and 8 units of NPH insulin at 7:30 a.m. to a client with a blood glucose level of 110 mg/dL. Which statements regarding the client's insulin are correct? 1. The onset of the regular insulin will be at 7:45 a.m. and the peak at 1:00 p.m. 2. The onset of the regular insulin will be at 8:00 a.m. and the peak at 10:00 a.m. 3. The onset of the NPH insulin will be at 8:00 a.m. and the peak at 10:00 a.m. 4. The onset of the NPH insulin will be at 12:30 p.m. and the peak at 11:30 p.m.

ANSWER: 2 The onset of regular insulin (short-acting) is one-half to 1 hour, and the peak is 2 to 3 hours. The onset of NPH insulin (intermediate act- ing) is 2 to 4 hours, and the peak is 4 to 12 hours. All other options have incorrect medication onset and peak times.

A nurse working on a telemetry unit is planning to complete noon assessments for four assigned clients with type 1 diabetes mellitus. All of the clients re- ceived subcutaneous insulin aspart (NovoLog®) at 0800 hours. In which order should the nurse assess the clients? Place each answer option into the correct order. _____ A 60-year-old client who is nauseous and has just vomited for the second time _____ A 45-year-old client who is dyspneic and has chest pressure and new onset atrial fibrillation _____ A 75-year-old client with a fingerstick blood glucose level of 300 mg/dL _____ A 50-year-old client with a fingerstick blood glucose level of 70 mg/dL

ANSWER: 2, 1, 3, 4 First, the nurse should assess the client with new onset atrial fibrillation and dyspnea. Diabetes increases the risk of coronary artery disease and myocardial infarction. Next, assess the client who just vomited. The client with a 300 mg/dL blood glucose level should then be assessed. This blood glucose level is not immediately life-threatening, but needs to be lowered as soon as possible. The client with the blood glucose level of 70 mg/dL can be assessed last because this is a normal blood glucose level. ➧ Test-taking Tip: Use the ABC's (airway, breathing, circulation) to establish the priority client.Then, look at the information provided for each client to determine the next priority. Consider that all clients received insulin at 0800 hours.

A client with asthma has pronounced wheezing upon auscultation. Suspecting an impending asthma attack, a nurse should: 1. have the client cough and deep breathe. 2. prepare to intubate the client. 3. prepare to administer a nebulized beta-2 adrenergic agonist. 4. have the client lay on his or her right side.

ANSWER: 3 A client with asthma who is experiencing wheezing and an impending attack is best treated with inhaled beta-2 adrenergic agonist drugs such as albuterol (Ventolin®). Oxygen and corticosteroids may also be used. Neither coughing and deep breathing nor positioning will stop the attack. Intubation is not effective in treating the underlying cause of the attack, which is an inflammatory response and would not be a first-line intervention. ➧ Test-taking Tip: Knowledge of mainstay therapies for asthma and emergency asthma management are essential. Note that the question does not indicate that the client is having a respiratory arrest, but rather the nurse suspects impending asthma attack. Do not read into the question or assume information.

A friend brings an older adult homeless client to a free health-screening clinic because the friend is un- able to continue administering the client's morning and evening insulin for type 1 diabetes mellitus. When advocating for this client, which action by the nurse is most appropriate? 1. Notify Adult Protective Services about the client's condition and living situation. 2. Ask where the client lives and if someone else can administer the insulin. 3. Contact the unit social worker to arrange for someone to give the client's insulin at a local homeless shelter. 4. Have the client return to the screening clinic mornings and evenings to receive the insulin injections.

ANSWER: 3 Advocacy focuses on the client's rights to make choices and decisions. The client has the right to receive and choose to receive treatment (autonomy). It is the nurse's responsibility to ensure that the client has access to health-care services to meet health needs. Adult Protective Services is an agency that investigates actual or potential abuse. Option 2 is data gathering and not advocacy. A screening clinic is not a permanent clinic where health-care services are provided.

A client with a suspected pulmonary embolus re- ceives a ventilation and quantification nuclear medi- cine (VQ) scan to evaluate regional lung ventilation of airflow and regional lung blood flow. In consulting with a physician, a nurse learns that there is a VQ mismatch. Based on this information, which action should be taken by the nurse? 1. Tell the client that tuberculosis treatment will be needed 2. Reassure the client that he/she does not have a pulmonary embolus 3. Explain to the client that further testing will be needed 4. Inform the client that the test was normal

ANSWER: 3 An imbalanced or mismatched VQ scan indicates some type of prob- lem with either ventilation or perfusion. Further testing is required, especially in the case of suspected pulmonary embolus. A chest x-ray, sputum culture, and Gram stain are used to diagnose tuberculosis; treat- ment should not be initiated. A VQ mismatch is highly suspicious, but not diagnostic of multiple lung diseases, including pulmonary embolus. A VQ mismatch is not a normal finding.

Which finding should a nurse expect when complet- ing an assessment on a client with chronic bronchitis? 1. Minimal sputum with cough 2. Pink, frothy sputum 3. Barrel chest 4. Stridor on expiration

ANSWER: 3 Barrel chest is indicative of a client with chronic bronchitis because of lung hyperinflation. Pink, frothy sputum is indicative of pulmonary edema. Minimal sputum with cough is more indicative of emphysema than chronic bronchitis, which usually is characterized by copious secretions. Stridor indicates some type of upper respiratory edema that would not be an expected finding in this scenario.

A nurse is preparing to admit a client with a confirmed case of tuberculosis. Which action is es- sential to infection control for this client? 1. Providing a positive-pressure airflow room 2. Wearing gown and gloves when handling the client's stool or urine 3. Using a National Institute for Occupational Safety and Health (NIOSH)-approved N95 respirator mask for staff and visitors 4. Keeping the client quarantined in the room until antibiotic therapy has been initiated

ANSWER: 3 Clients with a confirmed or suspected case of tuberculosis are gener- ally placed on some type of isolation precautions when hospitalized. These precautions include the use of high-efficiency particulate masks by those coming in contact with the client to prevent inhalation of po- tentially infectious respiratory secretions. The client should be placed in a negative-airflow room, which pulls the air out of the room and is vented externally. Gown and gloves may be appropriate if there is expected expo- sure to respiratory secretions. Although the client's movements around the hospital should be somewhat limited, the client can travel wearing a mask to reduce transmission risk and need not be quarantined.

A clinic nurse is evaluating a client with type 1 dia- betes who intends to enroll in a tennis class. Which statement made by the client indicates that the client understands the effects of exercise on insulin demand? 1. "I will carry a high-fat, high-calorie food, such as a cookie." 2. "I will administer 1 unit of lispro insulin prior to playing tennis." 3. "I will eat a 15-gram carbohydrate snack before playing tennis." 4. "I will decrease the meal prior to the class by 15-grams of carbohydrates."

ANSWER: 3 Excessive exercise without sufficient carbohydrates can result in unex- pected hypoglycemia. The food should be a simple sugar food because the fat content of a high-fat food will delay the absorption of the glucose in the food. Taking insulin or decreasing the carbohydrate intake prior to ac- tivity will lower the blood glucose level such that hypoglycemia can occur. ➧ Test-taking Tip: Recall that type 1 diabetes mellitus requires daily insulin administration and that activity increases energy expenditure and the demand for glucose.

A nurse is evaluating a client's outcome. The client's nursing care plan includes the nursing diagnosis of fluid volume deficit related to hyperosmolar hyper- glycemic nonketotic syndrome (HHNS) secondary to severe hyperglycemia. The nurse knows that the client has a positive outcome when which serum laboratory value has decreased to a normal range? 1. Glucose 2. Sodium 3. Osmolality 4. Potassium

ANSWER: 3 Extreme hyperglycemia produces severe osmotic diuresis; loss of sodium, potassium, and phosphorous; and profound dehydration. Consequently, hyperosmolality occurs. A normalizing of the serum osmolality indicates that the fluid volume deficit is resolving. A decrease in serum glucose indicates that the hyperglycemia is resolving, but not the fluid volume deficit. Serum sodium and potassium values should increase, not decrease, with treatment.

Two hours after taking a regular morning dose of Insulin Regular (Humulin R®), a client presents to a clinic with diaphoresis, tremors, palpitations, and tachy- cardia. Which nursing action is most appropriate for this client? 1. Check pulse oximetry and administer oxygen at 2 L per nasal cannula. 2. Administer a baby aspirin, one sublingual nitroglycerin tablet, and obtain an electrocardiogram (ECG). 3. Check blood glucose level and provide carbohydrates if less than 70 mg/dL (3.8 mmol/L). 4. Check vital signs and administer atenolol (Tenormin®) 25 mg orally if heart rate is greater than 120 beats per minute.

ANSWER: 3 Humulin R is regular insulin that peaks in 2 to 4 hours after adminis- tration. The client's symptoms suggest hypoglycemia, so a blood glu- cose level should be checked. The symptoms do not suggest a respiratory problem (option 1). Though diaphoresis, palpitations, and tachycardia are symptoms of both hypoglycemia and cardiac problems, the client had taken insulin 2 hours earlier. Treating the low blood sugar first will likely resolve the client's symptoms.

A nurse is caring for a client requiring positive pres- sure mechanical ventilation. The client has been fighting the ventilator-assisted breaths, and the client's blood pressure has been steadily decreasing. Which would be the most appropriate intervention by the nurse? 1. Place the client in the prone position 2. Notify the respiratory therapist to increase the positive pressure settings 3. Call the physician to suggest sedatives and paralytics 4. Prepare to administer intravenous aminophylline

ANSWER: 3 Positive pressure ventilation can lead to decreased cardiac output and a drop in blood pressure. Administering appropriate levels of sedation and a paralytic will allow the client to be more relaxed physically and mentally. This state of psychological and physical relaxation allows for greater synchrony with the ventilator. Adjusting the ventilator settings may assist in reducing client response to ventilation, but in many cases sedatives and paralytics are necessary. Placing the client in the prone posi- tion or administering intravenous aminophylline will not be a primary part of correcting the underlying issue.

A client learning about chronic obstructive pul- monary disease self-care at a community health class, asks a nurse why the participants are being taught about the "lip-breathing." The nurse should respond by explaining that pursed-lip breathing can help to: 1. reduce upper airway inflammation. 2. reduce anxiety through humor. 3. strengthen respiratory muscles. 4. increase effectiveness of inhaled medications.

ANSWER: 3 Pursed-lip breathing increases the strength of respiratory muscles and helps to keep alveoli open. It does not have an affect on upper airway inflammation, provide humor therapy, and is not a part of medication administration.

A client is admitted with diabetic ketoacidosis (DKA) associated with type 1 diabetes mellitus. The client's blood sugar is 320 mg/dL. The respira- tory assessment reveals respiratory rate of 32, with a deep, regular respiratory effort. Which acid-base imbalance is this client most likely experiencing? 1. Respiratory acidosis 2. Respiratory alkalosis 3. Metabolic acidosis 4. Metabolic alkalosis

ANSWER: 3 The elevated blood glucose level is a finding associated with DKA. Kussmaul respirations allow the body to "blow off" excess CO2 to compensate for the acidotic state and the decreased HCO3. DKA is a metabolic, not a respiratory, acid-base imbalance. As DKA implies, it is an acidotic, not an alkalotic, imbalance.

A client with chronic obstructive pulmonary disease (COPD) is in the third postoperative day following right-sided thoracotomy. During the day shift, the client has required 10 L oxygen by mask to keep his or her oxygen saturations greater than 88%. Based on this information, which action should be taken by the evening shift nurse? 1. Work to wean oxygen down to 3 L by mask 2. Call respiratory therapy for a nebulizer treatment 3. Check respiratory rate and notify the physician 4. Administer dose of ordered pain medications

ANSWER: 3 The night shift nurse should check the client's respiratory rate and report abnormal findings to the physician. Although uncommon, clients with COPD on high flow oxygen can lose their respiratory drive. Working to wean down oxygen by mask below 3 L will cause retention of CO2; oxygen by mask generally should be set at 4 L or greater. Although a nebulizer and pain medications may assist the client, the immediate need is to determine if the high flow oxygen is affecting the client's respiratory drive and to further determine the cause of the low oxygen saturations.

A nurse is helping a client with obstructive sleep apnea to apply a continuous positive airway pressure (CPAP) mask before going to sleep. The nurse knows that CPAP is intended to: 1. breathe for the client during sleep. 2. reduce intrathoracic pressure. 3. deliver high concentrations of oxygen. 4. prevent alveolar collapse.

ANSWER: 4 CPAP devices are intended for clients who can breathe on their own but need assistance in maintaining adequate oxygenation. The CPAP device keeps the alveoli open, allowing for maximal perfusion to occur. Although oxygen can be administered with a CPAP device, it is not always necessary. CPAP is not intended to reduce intrathoracic pressure.

A nurse is caring for a client with type 2 diabetes on a telemetry unit. The client is scheduled for car- diac rehabilitation exercises (cardiac rehab). The nurse notes that the client's blood glucose level is 300 mg/dL and the urine is positive for ketones. Which nursing action should be included in the nurse's plan of care? 1. Send the client to cardiac rehab because exercise will lower the client's blood glucose level. 2. Administer insulin and then send the client to cardiac rehab with a 15-gram carbohydrate snack. 3. Delay the cardiac rehab because blood glucose levels will decrease too much with exercise. 4. Cancel the cardiac rehab because blood glucose levels will increase further with exercise.

ANSWER: 4 Exercising with blood glucose levels exceeding 250 mg/dL and ke- tonuria increases the secretion of glucagon, growth hormone, and cate- cholamines, causing the liver to release more glucose. Exercise in the presence of hyperglycemia does not lower the blood glucose level (options 1 and 3). Administering insulin may be an option, but the blood glucose level should be known before sending the client to cardiac rehab (option 2). ➧ Test-takingTip:Think about the physiological effects of stress on blood glucose levels.Then eliminate options 1 and 3 because these are incorrect. Of the remaining two options, decide which action is safest for the client.

A nurse is assessing lung sounds on a client with pneumonia who is having pain during inspiration and expiration. The nurse hears loud grating sounds over the lung fields. The nurse should document the client's pain level and should document that: 1. lung sounds were clear upon auscultation. 2. fine crackles were heard upon auscultation. 3. wheezing was heard upon auscultation. 4. pleural friction rub was heard upon auscultation

ANSWER: 4 The client with pneumonia may have crackles, rhonchi, and wheezes as well as a pleural friction rub. A pleural friction rub has a distinctive sound that tends to be loud and grating and heard easily over the lung fields upon auscultation. Pleural friction rubs are also often associated with painful breathing. Fine or course crackles will have a moist, bub- bling, or Velcro-tearing sound. Wheezing tends to have a high-pitched sound.

A nurse is caring for a client diagnosed with respi- ratory acidosis. Which arterial blood gas findings should indicate to the nurse that the client's kidneys have compensated for the imbalance? 1. pH = 7.43; PaCO2 = 60; PaHCO3 = 35 2. pH = 7.35; PaCO2 = 50; PaHCO3 = 30 3. pH = 7.50; PaCO2 = 35; PaHCO3 = 30 4. pH = 7.44; PaCO2 = 45; PaHCO3 = 24

NSWER: 2 Respiratory acidosis results from increased PCO2. The kidneys respond slowly by retaining PaHCO3, which is a base. The normal PaHCO3 is 20 to 24 mmol/L. Full compensation occurs when the pH returns to the normal range of 7.35 to 7.45. Option 1 blood gas indicates a metabolic alkalosis with respiratory compensation. A pH of 7.43 is not an acidotic pH, and the body does not overcompensate. Option 3 indicates respiratory alkalosis with the kidneys responding, but the imbalance is not yet compen- sated. Option 4 is a normal blood gas finding.


Conjuntos de estudio relacionados

4.04: Applications of Coordinates

View Set

الحسبة والمحتسب في الاسلام مرحلة 1

View Set

BIBL 485 - Study Guide for Theology Review Exam

View Set

Structured vs Unstructured Data.

View Set

Chapter 1: What does assessment mean?

View Set

Bio lab EXAM 2 (Exercise 36,37,38b)

View Set

Principles Of Real-estate 2 Practice Test

View Set